Вы находитесь на странице: 1из 60

TU JANUARY 1990

F
@

Tlre sludnltt lnaUazine ol math and science


A smoothly varying family of lines in the plane almost
always forms folds and cusps. This family of lines was
generated as the set of normals to the curve that sweeps
through. Although this figure is a construction in the
plane, it gives a definite impression of three-dimen-
sionality. hr his article on developable surfaces, Dmitry
Fuchs offers a three-dimensional interpretation arising
from thebending of paper-seepage 16.
n
t_, UA TU JANUARY 1990

F EATU R ES
OUANTUM Artistic License Revoked
ll's leauiliful-{ut is il ssioltce?
by Albert Stasenko

t0 Geometry with a Trryist


Beltd his $[mt
by Dmitry Fuchs

24 Mathematics on the Fly


Tlle $ludefll msluill0 0l lllilI N[d sci0ll0s Pigems in ettery piUmnlule
by Alexander Soifer and Edward Lozansky
Cover afiby Ron Scott

A computer was used to generate 34 Low-Tem peratu re ph-ph-Physi cs


the sphere-within-a-cube on our [m $rulduid]ly of ltelillm ll
cover. You won't need a computer/
by Alexander Andreyev
though, to solve the foliowing prob-
lem based on that figure. Assume
the edges of the cube are line seg-
ments of no thickness and the
sphere touches the faces of the cube.
DEPARTMENTS
Let the length of an edge be 2 units.
Mentally place another sphere in
one corner so that it touches the
5 Pullisfiel''s Page

given sphere and the three faces of 0 letters lnom lle Editol,s
that corner. You could keep doing
this infinitely many times. If the
1 3 0uantum Smilus
radius of the original sphere tt = I, 28 l(aleidoscole
what does r,rro equal? If the diame-
ter of the original sphere dr=2,
3 0 llt llte lnl
what is the sum of the diameters of 33 Pnollem Col,nen
all the spheres that can be packed 38 lltnotlator$
into a corner? (Solution on page 55)
4l Bnainleasel's
42 Loo[inu Baclr
40 Al lhe Blacfiloal.d
5 0 lla[peninUs
5 3 $olutions

50 ClecImate!
0lliIllllll/JiruIry 1 000
Apublication of tbe
National Science Teachers Associatlon
OUANTUM Publisber
National Science Teachers Association
(NSrA)
17 42 C ortecticut Avenue NW
Bill G. Aldrtdge, Executive Director
Washington, DC 20009
202 328-5aOO USSR editor-in-cbief for pbysics and matbernatics
Bill G. Aldridge, Yuri Ossipyan
Executive Director Vice President of the USSR Academy of Sciences

and USeditor-in-cbiefforpbysics USeditor-in-cbiefformatbetnatics


Quantum Bureau ofthe Sheldon Lee Glashow William P. Thurston
USSR Academy of Sclences Higgins Professor of Physics Professor of Mathematics
3211 Gorky Street
Mellon Professor of the Sciences Princeton University
Moscow 103005 USSR
Sergey Krotov,
Harvard University
Execudve Director

US aduisory board I^arry Kirkpatrick


ln conjunction with tbe
Coach, Physics Olympiad
Amerlcan Association of
Lida IC Barett Professor of Physics
Physics Teachers (AAPI)
President, Mathematics Association of Montana State Univefsity
5112 Berqyn Road
America Bozeman, Montana
College Pa*,MD 2O740
Dean, College ofArts and Sciences
30r 345-4200
Mississippi State University Robert Resnick
Jack M. Wilson,
Mississippi State, Mississippi Professor of Physics
Executive Officer
Rensselaer Polltechnic Institute
George Bezsenyi Troy, NewYork
and tbe
Professor of Mathematics
Nattonal Council of Teachers
Rose-Hulman Institute of Technology Mark E. Saul
of Mathematics (NCTM)
Terre Haute, Indiana Mathematics Teacher
1906 Association Drive
Bronxville Schools
Reston, Yirginia220gl
Arthur Elsenkraft Bronxville, NewYork
703 620-9840
Coach, Physics Olympiad
James D. Gates,
Science Department Chair Barbara I. Stott
Executive Director
Physics Teacher Mathematics Teacher
Fox Lane High School Riverdale High School
Bedford, NewYork Metarie, Louisiana
Quantum is tbe autborized Englisb-
language uersion of Kvaoit, a physics and
mathematics maglzine published by the Judtth Ftanz
Academy of Sciences of the USSR and the
Professor of Physics USSR aduisory board
Vest Virginia University
Academy of Pedagogical Sciences of the
Morgantown, West Virginia Ylctor Borovlshky
USSR.
Deputy Editor-in-Chief
Don Holcomb Kvnnt magazine
Professor ofPhysics
Quantum would like to thank E:xon Cornell University Alexandet Buzdtn
Chemical Americas for its contribution.
Ithaca, New York Professor of Physics
Moscow State University
Special thanks also to Alex Mondale,
MargaretJ. Kenney
Catherine Lorrain-Hale, Peter Fish, and Amy
Professor of Mathematics dexey Sosinslcy
Stephenson for their help in launching
Boston College Mathematics Institute Associate Professor of Mathematics
Quantum. Designed by Ice House Graphics Chestnut Hill, Massachusetts Moscow Electronic Machine Dbsign Institute
and printed by Editors Press.

Managlng edttor Int ernatiana I cottsu I tant


Copyright O 1990 National Science TimothyWeber Edsrard D. Iozanstcy
TeachersAssociation. Subscription price
for t99O-91 (four issues) is S9.95; Production editor
inquire about bulk subscriptions. Elisabethd Tobta
Correspondence about subscriptions, This proioct was supported, in part,
by the
advertising, and editorial matters Adaertlslng manager
should be addressed to Quantum, National Science Foundation
Paul Kultzler Opinions expr$ed are those of th6 authors
1742 Connecticut Avenue NW, and not nocossarily th6e ot the Foundation
Washington, DC 2OOO9-ll7 l. Dlrector of publlcatlons, NSTA
Phyllis R. Marcuccio

0uanlum/Januat'y I 900
A
WIND OW ON
TOD,AM

Visions of what's coming. ln space.


Science. Technology. Your mind.
Science
with the National Science Tbachers Association
Gifted Young in Science
Theory... Potential Througflr Perform€ulce
ls there o promising student in your closs or in your fomily who might
love science? Gifted Young in Science, the first book in over 30 yeors
Giflred Young devoted to tolented science students, will help you encouroge them.
in Science Thirny-four scientists, teochers, ond scholors show you how to creote
f=-....-_ _-r-l the experiences ond environments thot encouroge ony child to un-
ttr--l ..]l[=' cover speciol tolents in the wide field of science. Steven Joy Gould,
nlr
U!!
!tr1-
I trl Iu
rilll lsoqc Asimov, Joshuo Lederberg, Lynn Morgulis, Glenn Seoborg, ond
,l other oword-winning workers in science offer the experiences which
t----
,t -]>l helped them choose their fields. And Gifted Young rn Science de-
Potential Through scribes methods-bosed on sound findings in philosophy ond psy-
Performance cholog5for odministering progroms for gilted students, designing
Hiio^
their curriculum, ond helping them excel. The NSTA creoted this
PaulI Bmndwein A Harr\,Passos volume to help educotors ond porents guide students to the excite-
I:l*::''
\.! k,\r n r!
..,:l*:Yx
\, r i!!Itrna\!r,{ ,,\ ment of science.
#PB-75/1, 1989, 422 pp. $24.00/hordbock
#PB-75/2,1989, 422 pp. $17.00/soft cover

Methods of Motion
An Infooduction to Mechanics, Book 1 ...Into
How do obiects move? lsooc Newton reolly believed thot on
obiect moving in o stroight line would continue with constont
Practice!
spd. Do your students? This monuol wos creoted to help teoch-
ers introduce the sometimes dounting subiect of Newtonion me-
chonics to students in the middle grodes. Ihe 27 octivities pre-
sented here use reodily ovoiloble moteriols to give students visuol,
ourol, ond toctile evidence to combot their misconceptions. And
the teocher-creoted ond tested modules ore fun: Morble roces, o
troctor-pull using toy cors, fettucini corpentry, ond film contoiner
connons will moke teochers ond students look fonvord to closs. F.

m*:';f;,:f :l;iiffi ttiru: j'['nff ffi ',ni;


useful for inservice workshops.
#PB39, 1989,168 pp. $t0.SO
ffiH All orders of $25 or less must be
prepoid. Orders over $25 must include o
purchose order. All orders must include o

.L\ .L\ffi.L\
postoge ond hondling fee of $2. No
credits or refunds for returns. Send order
to: Publicotions Soles, NSTA, l742Con'
necticui Ave. NW, Woshington, D.C.
20009.
PUBLISHER'S PAGE

Welcometo 0uanlum!
Arr or us at the National Science Teachers Association matics and currently serves as vice president of the Ameri-
are very excited about the premier issue of Quantum can Mathematics Society. In 1982 he received the coveted
magazine. Working with the American Association of Fields Medal for his achievements in mathematics.
Physics Teachers and the National Council of Teachers of The staff of Quanatm looks forward to working closely
Mathematics, and in cooperation with the Soviet Acad- with the members of the advisory boards, who are scien-
emy of Sciences, we're looking forward to producing a tists, mathematicians, and teachers of the highest caliber.
magazineof the highest quality. We expect Quantum to They'llprovide support in selecting translations from
provide interesting and stimulating material for the in- Kvant, soliciting original material, and reviewing it for
quisitive, bright young people of our country. When technical accuracy. From theirvantage points closer to the
Quantum contains equal amounts of material from both " action," they'll also help us tailor the magazine to fit your
the US and USSR, we hope it will be published in both needs and interests.
countries so that students around the world can share the
challenges and pleasures of Quantum's problems, articles, THr prcruRE BELow was taken at the celebrated
and other features. statue of Einstein on the grounds of the National Academy
Our new magaziae is extremely fortunate to have three of Sciences in Washington, DC. As the great physicist
top-notch scholars as its editors-in-chief. Academician gazes at the celestial map at his feet, he seems to draw us
Yuri Ossipyan is a physicisg vice president of the Academy into his " joy ardamazement at the beauty and grandeur of
of Sciences of the USSR, and editor-in-chief of our sister this world of which man can just form a faint notion . . . "
publication Kvant, the Russian-language magazine of physics The great Russian scientist and poet Mikhail Lomono-
and math {or secondary school students. He is also a sov/ a chemist of international reputation who helped
member of Congress of People's Deputies, a Soviet legisla- create Moscow University in 1755, was moved to utter
tive body similar in function to the US House of Represen- similar sentiments by a display of the northern lights.
tatives. Dr. Sheldon Lee Glashow is a professor of physics "Nature, where are your laws? The dawn appears from the
at Harvard University who has taught at summer science dark northern climes! Does not the sun there set up its
and math institutes for gi{ted high school students for the throne? Are not the ice-bound seas emitting fire? Behold,
last several years. In 1979he was awarded the Nobel Prize a cold flame has covered us ! Behold, the day has trod the
for physics. Dr. William P. Thurston, a professor of earth at night!"
mathematics at Princeton University, is also committed As he addresses nature in his ode, Lomonosov runs
to motivating academically gifted students. He has worked through a list of fairly technical questions: "What causes
closely with the National Council of Teachers of Mathe- bright rays to vibrate in the night? Why does a thin flame
strike the firmament? How does lightning without thun-
derclouds race fuom the earth to the zenith? How can it be
that frozen steam generates a conflagration in the midst of
winter? " He concludes by telling nature/ "What you say
about the things nearby is doubtful. So tell us, if you can,
how vast is the universe? What lies beyond the smallest
stars ? "
We hope this magazine will inspire in you the sense that
the pursuit of scientific truth is a grand andpever-ending
adventure.
Quantum is the just first of a number of cooperative
projects undertaken by US and Soviet science and educa-
tion organizations. We invite you to take part in some of
the student exchanges planned, and we hope your teachers
will participate in our teacher exchanges as well.
If you have comments/ suggestions, or ideas about this
premier issu e of Qu antum, please write to me. The next
issue will be out in the spring, and we'llbegrn publishing
Clockwise from top: Sheldon Glashow, Yuri Ossipyan, Bill quarterly in the fall of 1990.
Ndridge, Edwatd Lozansky, and NSTA president Hans Andersen, Aldridge
-BiIlG.
Iuantum/January I gg0
LETTERS FROM THE EDITORS

Ir cnrs MEgreat pleasure MarnEmarlcs AND pHysICs can be downright fun. I love
to introduce this first issue to think about mathemattcalptzzles/ quantum riddles,
of Quantum magazine to and the latest elementary particles, and I get paid for doing
the American reader. This it! If you become a professional scientist or a teacher, you
publication is the result of too may find it difficult to tell the difference between your
cooperation between Soviet hobby and your job-an enviable situation.
and American scientists and I wish Quantum had been around when I was a stu-
educators and is based on dent-it would have made it a iot easier for me to have
material {rom Kvant, a found fulfillment as a physicist. Although it has existed for
unique popular science some 20 years in its original Russian avatar, until now it
monthly magazine that has has been unavailable to American students.
circulated in the USSR for Almost every Soviet physicist I know either writes for
20 years. During this period Quantum
Kvant lwhichmeans "quan- or got into
tum" in Russian, as you physics by
must have guessed) has become the favorite mag azine of reading the
all Soviet high school students interested in mathematics, magazine
physics, and science in general. Its current readership is and doing
over 200,000. its prob-
Reading Kvant or Quantumisn't always easy, but if you lems. Hun-
really like science, I'm sure you'll find it exciting. Of dreds of
course/ the magazine contains recreational material, lively thousands
illustrations, humor, and amusing anecdotes from the of Soviet
history of science. But its core and main source of interest students are
are articles in physics and math that necessitate thinkin& avid sub-
sometimes pretty hard thinking to be understood. Experi- scribers to
enced Kvant readers sometimes even resort to pencil and theRussian
paperwhile reading the articles to work out equations or to version. Finally, an English-language edition has become
make their own sketches. Certainly a lot of brainwork and available to you!
some paperwork are required to solve the problems in our In this premier issue, we have translated some of the
Problem Corner, intended for those of you who like classic original Russian articles. In future issues, we plan
Ollmpiads and other probiem-solving competitions. But to publish contributions from scientists throughout the
all this work doesn't go unrewarded-few experiences are world. Read Quantum and enjoy it, and let us know how
as intensely exhilarating as the feeling "I've got it !" that we can make it even better'
comes as a flash when you've solved an intricate problem Lee Grashow
-sheidon
or grasped a profound idea.
My American colleagues and I hope that in the future Ase cHrro, I orrrN HATED arithmetic and mathematics
the cooperation between Quantum and Kvant will result in school. Pages of exercises were tedious and dull. They
in the simultaneous publication in English and weren't fun or challenging, they were
Russian of issues with almost the same content just a chore. I'dfind something else to
so that Soviet and American high school stu- entertain myself whenever I could. I
dents will be working on the same materials at doodled, I read books under the desk, I
the same timg competing peacefully and coop- stared out thewindow andlet my mind
erating in the spirit of the present rapproche- wander. Sometimes I tried to ptuzle
ment between our two great countries. Whether something out. Could you propel a
you have already developed an interest in math sailboat with a big fan on the boat to
or sciencg or have gathered from school courses blow on the sail? Might 0eventually
that these subjects are boring (as I did in my be periodic i{ you write it out in base 12
early teens), I hope Quantum will help you instead of base 10? How many ways are
discover the excitement inherent in mathe- there to fold a map into sixteenths, in
matics and the natural sciences. quarters each way?
Ossipyan When I became a mathematician I
-Yuri was surprised and heartened to find a

0llalltlllll/Janl|arI 1 Sg0
community of people who also take
pleasure in the kinds of things I enjoy,
who like to really dig in and try to
understand.
With the modem emphases on test
scores/ on "basics, " on mathematics
as a competitive sport, on getting n
" ahead" in math, and so on, it often
seems that the diversity,
liveliness, and depth of mathematics
richness., BringYour Potential To Power
Think logically! If you are an energetic and inquisitive studentwith
has been pruned away from the school
a desire for challenging experiences that are out ofthe ordinary -
e4perience. Mathematics isn't a palm then you belong at GW.
tree, with a single long straight trunk
. Learn from accessible frculty committed to your academic success
covered with scratchy formulas. It's a
. Have Washington, DC as your extended campus
banyan tree, with many interconnected
trunks and branches-a banyan tree . Experience unparalleled scientific internship opportunities with
that has grown to the size of a forest, goyernment agencies and prir,ate research corporations
inviting us to climb and explore. (If We offer B.A or B.S. Degree programs in mathematics, applied
you've never seen abanyan tree, it's mathematics, and physics through the Columbian College of Arts
worth going out of your way for.) and Sciences.
I have great hopes that Quantum For more information, write or call:
will open up a road to some of the Office ofAdmissions
breadth, wonder, and excitement of The George Washington University
math and physics. 2l2lI Street, NW
I'd like to post a waming, though, at Washington, DC2OO52
(2o2)9946040
the beginning of the road. Science
Gw is il equal oppomniry itulitution.
writing and math writing in particu-
|ar, tends to be dense and full ofhaz-
ardous turns and treacherous sand-
pits. When I was a child I took pride in
how many pages I read in anhour. In
college I leamed how foolish that was.
When reading mathematics ten pages DiscoverJohns Hopkins
a day can be an extremely fast pace.
Even one page a day can be quite fast. Discover the advantage of Johns Hopkins. As a world-renowned research
and teaching university, Hopkins has the advantage of attracting the very
On the other hand, if you already
best students who learn and work directly with the nation's best faculty.
understand something, you may get
more by skimming than by reading Discover:
everyword. You need tobe alert and r Hopkins'hallmark: support by Our new physics and astronomy
suspicious; you need to question and faculty mentors for original building with state-of-the-art
think about what you're reading in undergtaduate research at the research facilities
your own way. frontier of technology Hopkins' encouragement of
Quantum articles aren't written r The substantial undergraduate undergraduate enrollment in
like articles in scientiJic joumals, but access to the new Ardent-Titan graduate courses
some of the same reading habits still
mini supercomputer The two million volume Milton
apply. Don't be afraid to stop in midpara- r A low 8:1 student/faculty ratio S. Eisenhower Library
graph or midsentence when some- I Hopkins' commitment to the The Space Telescope Science
thing surprises or puzzles you. Speed undergraduate Institute
isn't the issue. Don't assume some- Return to: Office of Admissions, The Johns Hopkins University, Garland Hall, Baltimore, Marylaad 21278
thing is obvious just because an au-
thor treats it that way. What you work Please send me more information about the opportunity to discover Hopkins
out on the side, even though it takes Name
much more timg will have immensely Address
more value than whatyou read straiglrt
through.
High School Graduation Year
P. Thurston
-William Probable College Maior

0uInliltn/JrruEry I 000
ll's heaulilul-hul is it $cience?
There's something fishy about those waves

by Albert Stasenko

boat. They look like circles all components: one (V,,)parallel to one
I I
Jl :i: ;il,.,Ti1i,)X*','.1'?,,i'. centered at some point of the plane the
of the faces (sides of the boat) and
! ! river. Festively decorated boats containing the boat's waterline. Add other (Vr)perpendicular to it. the
So
Il with gaily coiored sails glide to these the position of the yard and flow of water relative to the side of
along the river. Their elevated bows sail relative to the boat's symmetry the boat really combines two move-
are decorated with dragon heads. plane. ments: it slips along the side at
The bright colors blaze in the Maybe you'll find something else velocitY V,, = Vcos o and crawls uP
sun. . The Vikings'voyage was that provides data on the direction on it at velocity Vr = Vsin cr.
long and dangerous." This is how V. and velocity of the boat, wind, and The perpendicular flow comprises
P. Knyazeva describes Nikolay Re- river. In the following sections, several layers: the lower layers dive
rikh's painting "Merchants from though, we'll concentrate on just under the huII (tig. 2h the upper
Overseas" (1901), which hangs in two phenomena related to waves on layer rises vertically, and we can
the Tretyakov Gallery in Moscow. water. easily estimate how high it reaches.
We'll make use of this striking Indeed, each water particle from the
poetical picture to ponder a few topics upper layer that possesses energy
in physics-the law of conservation The waue atthg lotlt mV ]12 and abruptiy changes direc-
of energy, dimensional analysis, and Why is the bow wave formed? tion can reach the heightJr where its
statistical analysis of experimental Suppose the water is flowing sym- potential energy mgh won't be
results. What has the painting got metrically at velocity V (the veloc- greater than its kinetic energy:
to do with all that? Well, I guess we ity of the boat relative to the water) Y2
only need it to liven things up a bit. around awedge (thebow of theboat) mgh<m+;
Seriously, though, don't you think with vertical faces forming an angle hence, z
it's a challenge to try and find the of 2cx (fig. 1). At the tip of the wedge
Y2
speed at which the Vikings aretray- we can split the vector into two p<L.
)o
elling just by looking at the paint-
ing? Relative to the moving water Therefore, by the law of conser-
and not the riverbanks, of course. vation of energy and the principle of
And what is there in the painting to superposition of motion/ we can
provide us with the necessary infor- estimate the boat's velocity: V,2 2
mation? First of all, there is the 2gh, which gives
wave formed on both sides of the
bow upon which the dragon rears its "2sht
head. And look at the circularwaves sin2g
moving on the water away from the
tigure I

8 ouantum/Jrnurry 1000
Surlace tllfltle$
All we have left is to measure the Surface waves are rather difficult
angle o and the height ft of the wave to investigate, but we can find a
at the bow. The relevant computa- great deal from very simple consid-
tion will be carried out below, but erations based on the physical quan-
first we'll consider the second avail- tities involved in the phenomenon.
able source of information on the First, we must elucidate the cause
boat's motion. of the phenomenon. The wave is a
traveling oscillation. And why does

0ulntum/ferlurs
the surface of the water oscillate (gI)1/2 wlll have the necessary units described in terms of the quantities
when its equilibrium state is dis- m/sec. Thus, u (m/sec|- [g (misec2) uo(m/sec), l"(m), o(n/m), p (kg/m')-
turbed? Any oscillation is the result l. (m)l t/2. uo denotes the velocity of waves due
of the interplay of two factors: iner- Notice that we've obtained a to surface tension, in contrast to the
tia leading to displacement from the proportion/ not an equality, since u in equation (2).
equilibrium state and the force re- any dimensionless factor k can stand Now that we have some experi-
storing the equilibrium state. in front of (g)")t/z-that is, u = ence in the use of dimensional analy-
If a "hump" appears on the wa- k(g)''ltP. We may have k = 0.5 or sis, let's deduce the expression for
ter's surface, then a restoring forcg perhaps k : 10. At this iuncture our the velocity of waves:
such as the gravitational force F^ dimensional analysis is powerless
o(kg/sec2)
proportional Io the acceleration of to help us, but it has already eluci- uo (m/sec) -
gravity g, cat bring water particles dated the main point-the physics p(kg/m3)xl,(m)
back to their equilibrium state (fig. of thephenomenon. The exact solu- (bearing in mind that 1 n = 1 kg m/
3a). Falling down, the hump wiII by tion of the problem leads to k = sec2). If we want to replace the pro-
its own inertia drop below its equi- I f (2n1trz-1'r^t ir, portionality sign with the equal sign,
librium state, another crest wiII be we have to take into account the
lr?t
forced out n earby,and so on. Conse- u=rii.
\rl 2n
(21 missing nondimensional factor.
quently, a wave specified by veloc- According to the exact theory, it's
ity u and wavelength l, (the distance And where is p (kg/m3)? It isn't equal to (Zn)rlz. Hence,
between humps)will move forward. there, simply because there is no-
In the case treated here, the density where to put the kg unit given by t ztr6' ,^
tfl
r

p of the oscillating water is a meas- p-it won't cancel out so as to provide 'o=V ),0
ure of its inertia. u with only the units m/sec. This is Isn't it remarkable that practi-
Thus, the propagation of a wave clear from the physical point of view: cally "free of charge"-without re-
on the suiface of a liquid is evalu- both the weight of the hump, accel- course to any physical theories-we
ated in terms of the following quan- erating it downward, and the mass have obtained the essential thing:
tities (with their units of measure- of the hump, determining its iner- the nature of the relationship be-
ment supplied): u (m/sec), I (m), tia, are proportional to p-hence, p tween the relevant physical quanti-
g (m/sec2), p (kg/m'). cancels out and density does not ap- ties! At this point it will be perti-
How are they connected? For in- pear in equation (2). (A similar phe- nent to recall the words of the great
stance/ how can we evaluate the nomenon occurs in the case of a
velocity u of the wave in terms of mathematical pendulum with string r (cmrsec)

the other quantities L, g, p2. Here the of length / whose period t = 2x


units used for the quantities given (?'lsll'doesn't depend on the mass
above will help us. for the same reason as above.)
We can see that among the units But if the waves become slight
for u there is sec-r. Among the other and ripple-Iike (later we'Il specify
three quantities only g contains the what "slight" is), the hump is pulled
time unit (namely, sec-2). Hence, u back to its equilibrium state by
- g,1,, and it is obvious that g has another forcg surface tension, which
played its role and can be of no depends on the coefficient of sur-
further use to us-it has provided face tension o (n/m). This is similar /0 V i{cm)
the required sec-1 for u. But at the to pushing a tallt rubber film with JE v'6
Same time 8'/' has given us m1/2, your finger. Because of the film's tiuurs {
whereas we need a " straight" m. So tension, a downward force Fo - o is
we must multiply 8tl2 by 1"1/2 to get exerted on your finger (fig. 3b). In physicist Enrico Fermi: "Physics is
everything in order-the quantity our case/ wave propagation can be no place for muddled thinking. . . .
Those who really understand the
nature of a phenomenon can obtain
fundamental laws from dimensional
thinking."
For ice-cold water/ o = 80 dynes/
cm= 8x 10-2n/m, p= 1 g/cm3= 103
kg/mt. Bearing this in mind, we can
plot the velocity of surface waves
defined by equations (2) and (3)
against Trtz(frg.4). The two curves
Figunc 3 intersect when l, = )vo 2 cm. So at
=

l0 0urnlum/January I 000
putting ourselves in a ridiculous
position-treating a f airy-tale paint-
ing as if it were an experimental
photograph is downright absurd.
Besides, we don't know such things
as the height of the bow, the size of
the shields, and so on. There's one
characteristic feature in the paint-
ing, however, that has apparently
changed very little since the days of
the Vikings: head size.
V=u Before reading on/ try to estimate
Vlu
h and l" yourself, if only with the
thuru 5 help of this "standard." This will be
very small wavelengths (1"<<\) wave or a bullet flies through the air faster your own personal "measurement. "
velocity is determined by surface than sound, distinctive accompany- Keep in mind, though, that each
tension and at large wavelengths ing "breakers" appear. The greater onlooker will have an individual
(i.rrl")by the pull of gravity. In the the object's speed V compared to the perception of the painting. Show
intermediate region (X X.) the ve- speed u at which the disturbances this painting to your classmates and
=
locity of surface waves is determined caused by the object propagate/ the ask each one about the values of 1.,
by both gravity and surface tension. closer these breakers are to the h, and a. (It's best to conduct the
The expression for wave velocity be- moving object. The three typical poll such that each student doesn't
comes more complex: u, = (u' * cases are shown in figure 5: if V < u, hear the answers of the others. In
tuoLltlz. The plot of u, versus 1,1/2 is the waves outrun the object and are this way the "readings" obtained
shown in figure 4 by the dotted line. merely condensed in the direction will resemble the independent read-
It often happens in physics that of travel; if V = u, the wave crests ings of a measuring device.) Record
it's much simpler to treat certain crawl on one another at one point (in the answers given, then plot the
Iimiting cases (1, approaches 0 and ), the direction of travel); if V > u, the values ofh along the horizontal axis
approaches infiniry in our case) than object outruns the waves andbreak- and, along the vertical axis, the
the intermediate region. Fortu- ers are formed. (Figure 5 illustrates number of students n who have put
nately, the intermediate region is of the third case.) forward the value h.
no concern to us because we can Looking at the Rerikh painting, I conducted such a poII among 30
safely state, iust by glancing at the we see that it iilustrates the first colleagues and obtained the graphs
Rerikh painting, that the circle case. There's no evidence that the shown in figures 7 and 8. It's inter-
waves cleariy have a wavelength waves in the direction of travel are esting that most estimates of h and
gteatfJ than \. Hence, the velocity compressed, we have to conclude )" were multiples of 5 c[l; Sorre wer€
of such waves is determined by equa- that the velocity of the boat is much multiples of L2.5 cm; but none were
tion (2). To find it we must less than that of the waves: multiples of a smaller number. (For
"measure" 1,, The necessary com- example, nobody put forward the
putations will be performed below, V << u. (41 valueft =27.1357891439 cm.) Itwas
but first we'll take another step. as if those polled were using a ruler
When awater bug runs on water Therefore, judging by the pattern of with a scale that reads down to 2-2.5
the circular surface waves/ the boat cm.
is practically standing still.
n(h)
Now let's turn to computations.
In order to perform them, we must
"measure" the height h of the wave
at the bow, the angle 2u, and the
wavelength i, of the surface waves.

ltleasut'emenls
We could try to measure the re-
quired quantities more accurately
fi[llf0 B, This photograph shows the by taking into account perspective,
breaker waves formed when a disk with h(cm)
a needle moves in a gas at a velocity
projection, foreshortening, and so
greater than the speed of sound-that is, on. But this is all very difficult and
the speed of wave propagation in a gas. time consuming. We would also be

0l|aItIlll/fealljls
With the help of these graphs, we The accuracy of the measure-
ments in our poll can be roughly
can calculate the averages <h> and
._"@
<1,> and regard them as sufficiently
fair.
estimated by the width of the smooth
curve at half its height (the red lines UT
= 0.7 (m/sec). (9)
The procedure for calculating the in {igures 7 and 8). A comprehensive
averages is standard. Each value h, theory of errors in physical experi- Now we come to a surprising
must be multiplied by the number ments has been developed and is result: equations (8) and (9) do not
of people n, who suggested it; then very similar to the one we've used. agree with equation (4). Indeed,
all values of I must be added and In an experiment/ of course, the poll according to the height of the bow
divided by the total number of par- is conducted among measuring de- wave, the velocity of the boat V is at
ticipants in the poll. As a result, we vices (not people). Ieast 2.3 m/sec; but if we base our
get And how about the angle a? judgment on the circular surface
30 Those polled agreed that 2a is be- waves (which are almost concentrj.c
\ hini 30 tween 90'and 180'(the bow of the in the painting), the velocity of the
<h>- i=t - L-\ h,n, boat in the painting is rather blunt). boat must be considerably less than
:9 30; Hence, the velocity of these waves u = 0.7
Lni m/sec, which is already less than V.
i= I sin45o < sincr< sin90o,

= 1 [(tx1o)+(2x12) \/2 If you pay enough attention to


30
+ (5 x 15)+ (7 x 20) + (4 x25)
!:<sinc< L lTl detail, you'Il discover physical in-
consistencies in other works of art
+ (6 x 30) + (2 x 40)+ (1 x 50) Now we've obtained all the data (which is probably due to the fact
+(1 x60)+(1 x70)l velocityof
necessary to estimate the t};,at art has its own aims and laws).
-7.!f-= 26 cm = 0.26 m. (5) the boat and surface waves. You can still admire the works of art
30 and derive aesthetic pleasure from
them, but you can also use them as
Similar1y, Tle fiesults attractive illustrations when dis-
Substituting the values of. h, ?,", cussing the laws of physics with
q),1= | KZ* 10) + (3 x 12) and sin o given by equations (5), (6), your little brother or sister. O
30 afi(71into equations (1)and (2l,we
+ (4 x 15)+ (1 x 18) + (8 x 20)
get Albert Stasenko, doctor of technical sciences,
+ (2 x 25) + (2 x 30)+ (l x 40) is a professor at the Moscow Physics and
+(3 x50)+(1 x70)+(1 x90)
+ (1 x 120) + (1 x 150)l
n="@
V ,lTtz
Technological Institute, wherc his specialty
is aerodynamics and gas dynamics, He en-
m. (6) = 2.7 (mlsec), ioys sailing in his sparc time.
= 34 cm = 0.34
2x9.8 (m/seC) x 0.26 (m)
These averages are indicated by 1
red arrows in figures 7 and 8. = 2.3 (mlsec);

z(I)
8

150 ),., ct',t

t0une I

12 0urnlum/Jrnurry 1
gg0
QUANTUM SMILES
twasiust
llng prohlgm all8r all0thgr
I failed to see the links
-but by B. M. Bolotovsky
things like this, doesn't know who
the author is. In addition to the
problems Dolgov told me, I'd heard
other problems of this kind; but no
matter how hard I tried, I couldn't
find who made them up. Person-
ally, I liked the problems so much I
decided to publish them. I hope you
enjoy them too, and I secretly hope
that one of you will be able to help
us find the authors. Or maybe the
authors themselves will see their
problems published and will step
forward. And now, without further
ado. . .

Problem 1. How many opera-


tions arc needed to put a hippopota-
mus into a refrigerator!
After Dolgov had posed the prob-
lem, I fell to thinking . . . Dolgov
came to the rescue. "I'11 tell you
how to solve the first probiem. You
have to perform three operations to
put a hippopotamus into a refrigera-
tor:
1. Open the refrigerator;
2. Put the hippopotamus in;
3. Close the refrigerator.lt
This solution helped me under-
stand the meaning of the word "op-
eration." Every action shown in the
solution was actually an operation.
Another problem followed.
Problem 2. How many operu-
tions are needed to put a giraffe into
the rufrigerator!
Having thought a little, I said,
"More operations are probably
needed to put the giraffe into the re-
frigerator than to put the hippopota-
mus there."
HE rruNc I wANT To rELL you is that I have no idea where
FrRST "Why?" Dolgov asked me.
these problems came from. I heard some of them from Oleg Dolgov. "Because the refrigerator won't
Soviet readers of our rnagazine have seen him on TV-for a few years hold the giraffe. The giraffe must be
he was the captain of a team on the very popular quiz show "What? folded up before it can be put in."
Where? When?" But even Dolgov, who can be considered an expert in "There's no need to fold the poor

In[[lrlll/Slllilos
animal," Dolgov said. "The refrig- I liked the problems so much that "why?"
erator's big enough. The giraffe can I memorized them and told them to "Because he already broke one
easily fit into it if it's empty." my daughter Katya when I got and he only has two."
"So it's e4ough to perform three home. She's in the sixth grade. To Then Katya and I decided it
operations, as before: open the re- my great surprise, Katya solved the wasn't very good of us to pose the
frigerator, put the giraffe in, and problems in no time, one after the problem about the boy because we
close the refriger ator." other, and gave me another problem felt sorry for him-it's very painful
"Wtang," Dolgov said. "Four to solve. to break a leg. So we changed the
operations are required." And he Problem 6. A boy fell down from problem so that it was about a
enumerated them: the fourth step of a staircase and chair-a chair falls down the stairs
1. Open the refrigerator; broke his leg. How many legs will and breaks its legs. We felt sorry for
2. Take the hippopotamus out; the boy break if he falls from the the chair too, though not as much as
3. Put the giraffe in; fortieth step! for the boy. Besides, the problem
4. Close the refrigerator. Not sure of the answer/ I said, si:emed very interesting to us. One
Need I explain? I'd forgotten the "Forty steps . are ten times as chair isn't too much to sacrifice for
refrigerator was full. The hippo- many as four steps . . . so the boy a problem like this.
potamus was still there after the should break ten legs. But that's
first problem. probably wrong." WEIL, I'vr roln vou all the
And the problems kept coming. "It is," said my daughter. "chain problems" I know. Do you
Problem 3. The hippopotamus "What's the right answer?" know any others? If so, send them
and the gfuaffe are akilometer away "The boy wili break only one to Quantum and amuse your fellow
fuom a river. Which of them will leg." readers. o
rcach the riverbank ffustl
When solving problems like this,
it's useless to think. Nevertheless,
I thought a little before saying, "The NE oF Eocen Arrau PoE's
giraffe will get there first-its legs lesser-known "Tales of the
are longer." Crotesque" is entitled "The
"Wrong agairr," said Dolgov.
"What's the right answer thenl"
The Unparalleled Adventure of One
Hans Pfaali." The hero of the tale
"The hippopotamus will reach makes a remarkable discovery-he
the river first." obtains an extraordinary gas whose
"Obviously! Why?"
"Because the giraffe is still in the
aduenlul'e $ OI density is 37 .4 times less than that
of hydrogen. A balloon containing
refrigerator." such a gas possesses an incredible
I laughed and decided to keep lift force. With the help of this bal-
trying my luck.
Problem 4. How many hippo-
Hans Pfa a I loon, Poe tells us, Hans Pfaall suc-
ceeds in reaching the moon.
potamuses can a five-ton truck It's common knowledge that
hold! there is no gas in nature that's
I started to think again, but lighter than hydrogen-no proof of
Dolgov didn't let me think for long. that will be given here. {But can you
"Don't waste your time. I'll tell explain this fact?) Nevertheless, the
you the right answer. It'lIhold five following problem is of interest: if
tons of hippopotamuses-a full such a gas existed, by what factor
load. Solve the next problem your- would it increase the lift force of the
self-and be quick!" balloon (as compared to hydrogen)?
Problem 5. How many gtuaffes Despite the simplicity of the
can the five-ton truck hold!
"Five tons of giraffes," I said, not
The deflating physics f:_oln::::n*: ral to give a
quite sure of myself. or two st,andJii1ioi ::[T,ffiT:X1f::"* ;ffl:il;;
"Wrong! Not a single one." since the gas is lighter than hydro-
"why?" gen by a factor of 37.4, if follows
"Because the truck is fulI of hip- that its lift force is greater by the
popotamuses.// by V. Nevgod
same factor. Perhaps Poe counted
Sure enough-after problem 4 the on such a hasty conclusion by read-
hippopotamuses were still in the ers when he wrote his tale. Then
truck. No one had taken them out. again, it's equaliy possible he him-

14 0rrrlulll/JaruarJ I 090
self made that same error by follow-
ing a seemingly logical pattern of
thought.
Simple calculations, however,
show that the gain in lift force is so
small that it should be considered
negligible. We'll prove this by find-
ing the lift force of a hydrogen bal-
loon and that of a balloon contain-
ing Hans Pfaall's gas.
Consider a balloon with a vol-
ume of I m3. The density of air is
0.00129 glcm'i of hydrogen- gas-0.0000O24 glcm3. The lift Donning lead-soled boots in addi-
0.00009 g/cmt; and of Hans Pfaall's force of a balloon is equal to the tion to his lead suit and carrying a
difference between the buoyancy bag of solid lead, he's able to go
force (equa1 to the weight of the air outside again like a normal person.
displaced by the balloon) and the The obvious question pops up:
weight of the gas contained in the how much lead does Pyecraft need
balloon (the shell of the balloon to walk around? Let's perform a
being considered weightless). The simple calculation. Suppose fatty
lift force of a hydrogen balloon, Pyecraft weighs 1,000 newtons (his
then, is about 12 newtons; that of a mass being 100 kg). His body vol-
balloon containing Hans Pfaall's gas ume would then be about 0.1 m3.
is 12.9 newtons. On becoming weightless, poor
So the gain in lift force is only Pyecraft sort of turns into a
about 0.9 newton! The result is so strangely shaped balloon of the
insignificant that it wasn't worth same volume. Its lift force would be
Hans Pfaall's (or Edgar Allan Poe's) only about 1.3 newtonsl
effort to discover the miraculous So what's the real "truth about
superlight gas and violate the laws Pyeuaft"? Even in his ordinary
of nature to boot. (To be fair, we clothes, tryecraft didn't have to float
should remember that the Mendele- under the ceiling of his study-he
yev periodic table had not yet been could sit at his desk (though not too
drawn up in Poe's lifetime.) What steadily) and could even walk care-
led to the blunder? The low weight fully around the room (avoiding
of hydrogen. abrupt movements). He'd have no
Were it possible to obtain a gas need of a lead suit or lead boots-he
lighter than hydrogen by a factor of could go outside in his regular
1,000, it wouldn't significantly help clothes, maybe with the addition of
boost a balloon's lift force. The a heavy briefcase to protect him
limit of such an increase is 0.9 from a strong wind. He'd be in no
newton-the weight of the hydro- danger of floating up into the upper
gen itself. atmosphere.
It turns out that Pyecraft's "air-
H. G. Wprrs wrote a popular fan- worthiness" and the problems he
tasy called "The Truth About faced were greatly exaggerated. It's
Pyecraft." In it a funny-looking fat doubtful, of course, that H. G. Wells
man/ named Pyecraft, desperately failed to notice this when writing
wants to lose weight. He takes a the story. Most likely, he deliber-
mysterious Indian remedy and loses ately ignored the relevant data so he
more than his excess weight-he could depict more colorfully and
loses a1l his weight! Day after day eloquently the comic misadven-
he floats under the ceiling of his tures of the hapless Pyecraft. No
study, not daring to go outdoors- doubt the author was convinced
he's afraid of floating away like a that his readers, carried away by the
balloon. Eventually someone ad- brzarre plot, wouldn't spot the exag-
vises Pyecraft to order a special suit gerations. O
with lead sewn into the lining.

0uanlum/Smiles
AKE A SHEETOF PAPERAND BEND IT
without crumpling it. You get a
piece of a surface whose shape
depends on how you bend the
sheet. Some possible shapes are

Bend lhis $hesl shown in figure 1.


It's certainly not true, though,
that any surface can be formed by
bending a sheet of paper. For ex-
But do not fold, staple, or mutilate ample, it's common knowledge that
you can't bend a sheet of paper into
a sphere-if you press a piece of
by Dmitry Fuchs paper onto a globe, folds necessarily
appear. Of course, you can roll a
sheet of paper into a tube or Paper

thum I

18 lluarttltn/JlttrrrY 1000
say/ and we have to agree. It's not
that there's anything wrong with our
definition. It's just that this finicky
reader shouldn't bother to read on-
we won't be offering any rigorous
definitions or proofs here.
Let's just say that all our proofs,
as in an ancient manuscript on ge-
ometry from India, will pretty much
consist of one word: "Look!"
However, for the perceptive reader
(not the pedantic one, who has
closed the magazine by now) we can
add that two essential physical prop-
erties of paper sheets are assumed
here. The first rs unstretchability
(incompressibilityl, which means
that a curve drawn on the sheet can
change its shape when the paper is
bent but must always preserve its
length. The second property is flexi-
bility, which means that there are
no other constraints on the nature
of the bending.
." ' :: .
:1..

[::,:;:::'.,'
,' 'r:. ,- : . I '
Tsr recr rHAT not al1 surfaces
are developable is evident from the
Iact that every developable surface
is a ruled surface. This means you
\g
v
can place a knitting needle any-
ves : .,:.,:::
where on the surface so that the
ti:i:t::r:::i::,:,'l:':"-,r::rri::,:iirtrt!:t: t,.-,.r]r,:il:ii:::r,:::t:l
:
needle touches the paper along an
r.li:.:r"':: r',:.l:i.:l::::i::r.lri:rl :l:''j:::,::i-,:::.::rr, ,:::,
r ,:.r::::r:::,:.'lirt:f entire line segment containing any
::lt1;1:r::1-,:,:rr:::1.1,;;::1.lii:t,it:.::r::: ir ,:l:t::i.rr,.l:.':..i'::: :. :: l:..1 chosen point. (Proof: try it, as in
figure 3.) In other words, a develo-
pable surface consists of straight
line segments that are contained en-
tirely within it. These segments are
calied the linear rulings (or simply
the rulings) of the surface.
If some point of a developable sur-
face is an internal point of two dif-

tlgurc 2 tigut.s 0
it up
cone, but it's impossible to furl ferent rulings, then a whole section
in four like a handkerchief without of the surface near this point is flat
creating folds (fig. 2). (fig. al. We'll exclude this case from
DprrNrrroN. Surfaces that can be consideration and require that no
rcpresented as bent sheets of paper part of our surface/ no matter how
are called developable surfaces. small, be a piece of a plane.
"That's not a definition at aII, Therefore, exactly one ruling
that's nonsense-nothing useful can passes through each point of our
come of it," the pedantic reader will surface. These rulings form a con-

0urlllllln/[Erllllc 11
surface, however, all such perpen-
diculars must belong to the same
plane. In other words, not only can
you place a knitting needle on a
developable surface at any given
point, you can lay the flat of a ruler
tangent to the surface along the
same line as well (fig. B). (Try it and
. see.) This property of developable
surfaces is sufficient-that is, any
surface possessing this property is
necessarily developable.
By the way, have you ever come
across a hlperboloid of one sheet, or
a hyperbolic paraboloid (fig. 9)? If
not, no matter-we don't need them
tiUune 7 here. But if you're familiar with tiguru l0
tinuous family of segments sweep- them, notice that they arc not de- sheet of paper, bend it approxi-
ing out the surface (fig. 5). Some of velopable. Why not? Because a pair mately as shown in figure 5, and try
the segments can degenerate into of rulings passes through each point to imagine the shape of the surface
points on the boundary of our piece of these surfaces, which is impos- obtained by extending the rulings in
of surface. (This last remark is sible for developable surfaces. the direction where they converge
aimed at the shadow of the pedant (fig. 11). Put the magazine aside and
hovering in the background.) really think about it. Then
come back and look at the
Now you sHouroN'T THINK answer.
that every ruled surface is The answer is this: the
developable. There are plenty surface will no longer be
of ruled surfaces-a line seg- smooth-a cuspidal curve
ment moving in space sweeps will appear (ftg. l}a). (If you
out a ruled surface, for in- guessed it on your own,
stance. you're a true geometerl) A
Let's take any ru1ed surface, cuspidal curve is a curve in
make a linear ruling, and for whose neighborhood the pla-
each point of the ruling draw nar section of the surface
the line perpendicular to the looks approximately as
ruling and tangent to the sur- shown in figure 12b.
face (fig. 6). We'll obtain a pat-
tern similar to a Christmas I pRoursro
ro avoid proofs.
garland (fig. 7)-our perpendiculars tigurc 0 It's not that I dislike proofs (that cer-
protrude haphazardly as they turn tainly isn't true). And it's not be-
on the ruiing. For any developable Loor er FTGURE 5 again. Recall cause the proofs of my statements,
that we're not dealing with an infi-
nite surface but only a piece of it.
(Indeed, a sheet of paper can't be in-
finitel) The piece is delineated by
the rulings. What will happen if we
try to extend them in one direction
or another?
At first the question seems quite
innocent. If you extend the rulings
of the surface in figure 5 "r)p" toward
where they fan out, nothing interest-
ing will happen-the surface will
just grow, becoming LTatter and flat-
ter (fig. 10).
But what if you extend the ruIings
in the opposite direction? Take a
fhum I tigure 1 I

l8 0llalllllll/Jrrrary 1gg0
tiilro 12 tigu'o 13 tigul'e 14

and of my last statement in particu- lines accumulate around a certain velope." (A similar theorem is true
lar, ate so complex they can't be ex- curve to which they're all tangent for a family of curves.) So we have
plained to a high school senior who (fig. 13). Now, a very simple theo- "catrght" the cuspidal curve.
knows a little calculus (that's true "But wait a minute," you (the
only to a certain extent). It/s just perceptive reader) will object. "You
that the proofs of the theorems in can apply this argument (if it de-
this article-at least the proofs that serves that name) to any ruled sur-
I know-involve playing around face. For example, the one-sheet
with formulas: defining the surface hyperboloid (fig. 9) has a line to
by an equation, expressing develo- which the rulings projected onto the
pability in terms of derivatives, and picture plane are tangent (the con-
so on and so forth. I don't want to tour hyperbola), but there's no cus-
present a proof that isn't likely to pidal curve on this surface."
clarify anything. But I still have to You're right, as always. But I
convince you that certain state- have an argument stashed away that
ments are correct even if I don't will probably convince you. The
prove them, curve you have in mind is the edge,
Take one more look at figure 5. or visible contourt of the depicted
As with any depiction of an object surface. At each point on this curve
in space, that object is projected onto tlgure 15 the plane tangent to the surface is
a plane (for example, onto this maga- perpendicular to the picture plane.
zine page). Thus, we have a family rem in calculus states that any con- (See figure 14, where the visible
of lines on the plane. Copy it on a tinuous family of lines, unless contour of a sphere is shown.) But
separate sheet of paper and extend all they're all parallel or pass through a for a developable surface, the tan-
the lines. You'll find that these fixed point, will have such an "en- gent plane is one and the same at all

tigum 18

0urnlum/fealune 1g
fi uuord or luuu lnoro ahnul
deunls[nile surlace$
(and a model to help visualize them)
"How PAPER BENDS" sounds at swept out by the line segment as it store, but stiff wire like coat hangers
first like the most mundane (and in- moves along the helix. Fuchs shows or bicycle spokes should also work.
nocent) of topics, but Dmitry Fuchs that this surface has a sharp crease/ Cut out circles about size of a pea-
shows how intriguing (and tricky) or "cuspidal curve,'f along the helix. nut butter jar lid from the middie of
an ordinary sheet of paper can be. This is weird. How can a straight two sheets of paper. Throw away
As I was reading his article I wasn't line sweqp out a surface with a ereasel the circles and join the sheets along
content to bend paper aud imagine Computer pictures help a little. the edges of the holes. To do this,
in my head what happens as the The figure below shows schemati- lay lengths of tape flat along the
lines are extended, even though I cally what happens. The sinusoidal circle so that they overhang the hole.
knew something about developable curye weaYing up the center is the Cut slits from the edge of the tape up
surfaces from studying differential projection of the helix (cos t, sin t, t) to the circle, fold the flaps of tape
geometry. It',s hard to visualize the onto the x-z plane. The dotted curves under, and squeeze together.
cuspidal curves of a developable are a cros$ section of the surface Cut slit in both sheets from the
a
surface, let alone a swallowtail. I swept out by the lines tangent to the circle out to an edgeso you'll be able
made some models and some com- helix. You can see the cusps along to open the model later to form a
puter pictures-maybe you'd like to the helix. Can you visuaiize this? It helix.
try some for yourself. really is strange. Now, one by one, lay the thin,
The simplest spatial curve that A convincing demonstration can straight sticks between the two sheets
doesn't lie in a plane is a helix (like be made with paper. You need thin, of paper so that they nestle against
the coils of a spring|. Imagine a straight sticks of some type to con- the circle where they're joined. Tape
straight line segment taqgent to the trol the direction of the rulings. I the clockwise end of each stick to
helix, touching it at the midpoint of used wooden skewers {like long the lower sheet of paper and tape the
the segrnent. Now imagine the sudace tocthpicks) from the local grocery counterclockwise end to the upper
sheet of paper. With about
six or eight sticks the model
will have reasonabie stiff-
ness.

The sinusoidal cuwe in the


centil of the illustration at
left is a helix (for example, a
spring) as seen from the side,
If you take a line tangent to
the helix and move it along
the helix so that it rcmaini
tangent, it sweeps out a
surface. This surface is an
example of a "developable
surface." one that can be
modeled by bending sheets
of paper in tfuee dimensions.
The dotted curves on either
side of the helix ate uoss
sections of the surface. This
surfa ce interp enetr at ed it s elf
and has a sharp cusp, or
crease, where it meets the
helix,

20 0mrlum/Jruurry 1000
Now open the model so that the
circie becomes a helix, the sticks be-
come lines tangent to the helix, and
the paper becomes a developable
surface swept out by the tangent
lines of the helix. You can stretch it
so that the helix is very long or
flatten it until the helix is very shal-
low. You'Ilsee the cusp where the
two sheets join. The cross section of
the model is like the figure at left.
Despite all the sticks, the model
stiil has considerable flexibility. If
you know about the curvature of
curves in space, the circle can be
bent to the shape of any other curve
with the same curvature.
You can form additional turns of
the helix, if you like, by joining
more sheets of paper to the first two.
You end up with a large paper screw.
It will look nicest if you trim each
sheet of paper to be a circle concen-
tric with the hole and cut the wooden
The figure above is a close-up of the figure on page 20, showing how the cusp
ribs so that they just reach the outer
becomes arbitarily sharp as the scale gets finer.
circle at both ends.

|usr ron rHE RECoRD, I tried a


quick model of the swallowtail, but
I'm not proud of the result. Try it for
yourself! If you find a good con-
struction technique, I'd like to hear
about it.
P. Thurston
-William

The family of lines at ilg,ht is the set


of normals to the curye (t4 - t2, tl
weaving through the center of the
figure. You can see the folds where
the lines are tangent and the cusps
where the folds reverse direction.

0ffi [lIm/tcrlllrs 2l
[iUure 2l

t0ut,u l0 tiguru 20 fhum 22


points on the ruling (remember the nonconical developable surface can path perpendicular to the rulings
ruler lying flat on the surface!), and be obtained in this way (Euler's (fig. 17). Since rulings are tangent to
it's obviously not perpendicular to theorem). the cuspidal curve, we'll either
the picture plane at points that quickly approach or quickly move
don't belong to the hypothetical away from it. What might the tran-
cuspidal curve. This means that the sition between the two states con-
tangent plane isn't perpendicular to sist of? Look at figure 18: a sheet of
the picture plane at the points on papett rulings, and two segments of
this curve either-it's positioned as the cuspidal curve. But what's be-
shown in figure 15. This undenia- tween the segments? A smooth
bly proves that our curve is a cuspi- curve like the dotted line in the fig-
dal curve and a visible contour isn/t. ure? No, that would be too incred-
We can look at this from another ible-a curve like that can't be tan-
angle. A developable surface (whose gent to the rulings at every point. So
rulings arert't parallel and don't aII only one possibility is left: the cus-
pass through one point) consists of pidal curve itself must have a cusp
straight lines tangent to one curve- (fig. 1e).
the cuspidal curve. So you can con- How is the surface near this in-
struct a developable surface by tak- conceivable point structured? Let's
ing a spatial curve (with no flat tiguru 23
start with a picture. The surface
stretches) and drawing all its tan- itseif is shown in figure 20. In ad-
gents (fig. 16). These tangents will Is trar alr? No, as you'Il soon dition to a cuspidal curve, it must
sweep out a developable surface, and see. Let's mentally enlarge the de- have a self-intersection line. Figure
the original curve will be its cuspi- velopable surface until we're able to 2l shows several parallel planes
dal curve. Any noncylindrical or walk along it, and let's choose a cutting through this surface. To

r,ril. !] ii:ili iti,:: ir


iir;f111

tigum24

22 0rrttltttn/Jattttany I 000
convince ourselves (to some extent/ such interesting geometrical ramifi- equation has 4 solutions;
at least) that the surface really does cations? Now you know why the - on its boundary, except for the
look like that, we'll do what we did swallow is perched at the beginning cuspidal curve and the self-intersec-
with the Euler theorem-we'll take of this article! tion line, it has 3 solutions;
the hypothetical cuspidal curve and - on the lines, except the vertex,
draw all its tangents. IN coNCLUSToN I'11 add a few it has 2 solutions-the same num-
To construct a "typical" spatial words about the swallowtail itself. ber as for any point lying above the
curve with a cusp/ take a planar This surface appears quite fre- surface (and not in the box);
curve with such a point and bend its quently in three-dimensional ge- - on the whole surface, except for
plane a bit (fig. 22). Now draw all the ometry/ and many natural problems the boundary of the box but includ-
tangents to this curve and look at the in calculus and mechanics are re- ing the vertex, it has 1 solution;
result from above. Divide each tan- lated to it. However, the picture - below the surface, there are
gent into three parts as shown in fig- (though not the name, which was no solutions. O
ure 23. Now draw the first, second, not coined until the 1950s by the
and third parts of all the tangents famous French mathematician R.
separately (fig.2q. In the first pic- Thom) first appeared on the pages of
ture we get a slightly flexed upper l9th-century algebra textbooks in
membrane stretched between the the following context.
Dmitry Fuchs is a graduate of Moscow {Jni-
two branches of the cuspidal curve. Consider the equation xn + arxn-z vercity, where he now works as a leading re-
In the second picture a two-piece * ar-x"-t +... + an_t=0.Thenumber search fellow in I. M. Gelfand's math and bi-
joint spanning the branches of the of solutions canvary from zero to n. ology laboratory. Fuchs has published a

cuspidal curve and the self-intersec- For example, the equation:.J + ax + dozenbooks and over 100 rcsearchpapers in
algebraic topology. He has wiltten many
tion line will appear. And finally in b = 0 can have 1, 2, or 3 solutions.
articles for Kvant and has been very active
the third picture we'll get the rest of In order to find the actual number, in math Olympiads (he is a former winner
our surface. (Notice that the figures we have to draw the discriminant himself ). In his free tine Fuchs enioys poetry
drawn in the second and third pic- cuwe 4a3 + 27b2 = 0 on a plane with and painting.
tures have a "fracture" along the coordinates (a,b) (fig. 25lr. lt the point
self-intersection line.) The entire (a,b) belongs to the interior of the
surface is called a swallowtail-it shaded domain, the equation has 3
really looks like one, doesn't it? solutions. If it lies on its bo.undary
So we see that an arbitrarily bent (exception for the cusp), the equation READINGS FOR
(but not crumpled) sheet of papet, has 2 solutions. In the remaining ENRICHMENT IN
after infinite extension of its linear cases it has 1 solution.
SECONDARY SCHOOL
rulings, turns into a surface with a An analogous problem for the MATHEMATICS
cuspidal curve that itsel{ has at least fourth degree equation / + a* + bx
Edited by Max. A. Sobel
one cusp. Near each cusp this sur- + c = 0 leads to a swallowtail, simi-
face looks like a swallowtail and has lar to the one in figure 20, located in Academically talented students, this
book will expand the mathematics
self-intersections. space with coordinates (a,b,cl. If the curriculum for you! (lt is also appro-
Would you have imagined that a points (a,b,clhe priate for use by students of varying
garden-variety sheet of paper has - inside the trihedral "box," the abilities.) The material contained has
something mathematically exciting
for everyone. This book will save you
time; it is a compilation of articles
from the Mathematics Teacher, En-
richment Mathematics for High
School (NCTM's 28th Yearbook),
and Topics for Mathematics Clubs.
It also features three new chapters on
the harmonic mean, rotation matrices
and complex numbers, and how
computers and calculators pedorm
arithmetic. 1 988; 298 PP.; #374; $1 1 .

lEEl
qP
NATIONAL COUNCIL OF
TEACHERS OF MATHEMATICS
1906 Association Drive
Reston, V422091
(703) 620-9480, FAX (703) 476-2970

figure 25

llIaIlllll/tsrll|IE
1

ffi
$
t'
Solution: Denote the given inte-
gers by a1t a2t . . . , an. Define:

St=d'
Sr= ar+ a,

Piueolt$ in stlel'y :

Sn= ar+ a2+ "'t ar'


piUeonhole If one of the numbers 51, 52, . . . , S,
is a multiple of n, we're done. Assume
now that none of the numbers S,, S,
. . . , S, is a multiple of n. Then all
. . . and NewYorkers, and chess possible remainders upon the division
pieces ., ,
of these numbers by n are 1,2, . . . ,
n- l-that is/ we get more numbers (n,
which are our "pigeons") than pos-
by Alexander Soifer and Edward Lozansky sible remainders (n - 1, which are our
"pigeonholes"). Therefore, among the
numbers 51, 52, . . . , Snthere exist two
HE PIGEONHOLE PRINCIPLE/ ALSO Let's take a look at some problems/ numbers-say, So and So*,-that give

l known as the Dirichlet principle


after the famous mathematician
from easy ones to those that may be
not so easyformanyof you.
the same remainders upon division by
n.
Peter Gustav Dirichlet ( 1805-1 859), We are done, because
plays a special role in mathematics as 1. New York City has 7,100,000
well as in problems at mathematical residents. The maximum number of ( 1 ) Sft., - Sk is a multiple of n;
competitions. The principle is very hairs that cangrow on ahumanhead is (2) So.r-SL=4L*, !ak*zl ...tak*r.
simple: Ifkn + 1 pigeons (k andn are 500,000. Prove that there are at least
positive integers) are placed in n pi- 15 residents of New York City with In other words, we found some of the
geonholes, then at least one of the the same number of hairs. given numbers-namely, a1,*1, a1,*2,
holes contains at least /< + I pigeons. Solution: Let's set up 500,001 . . . , ak*tt whose sum is a multiple of n.
I
It's very easy to prove this. pigonholes labeled by integers 0 to
Assume that there are no holes that 500,000 and put residents of New York 3. Given a real number r, prove that
contain at leastk + 1 pigeons. Then into the holes labeled by the number among its first 99 multiples r, 2r, . . . ,
of hairs ontheirheads. Because Z1m,0m 99r t}rerc is at least one that differs
the lst hole contains k pigeons
< > 14 x 500,001 + 1, we conclude by the from an integer by not more than
the 2nd hoie contains k pigeons
< pigeonhole principie that there is a pi- 1/100.
geonhole with at least 14 + 1 "pi- Solution: Let's roll the number line
:
geons"-that is, there are at least 15 on a roller with a circumference equal
the nth hole contains < k pigeons residents of New York with the same to I (fig. 1). All integers will coincide
{_ number of hairs. on the roller with zero. Now we
divide the circumference into 100 arcs
E=-:

E
E 2. Given n integers, prove that one of ec1ual length lh1.2l. If at least one of
z) This contradicts the given fact that
there are kn + L pigeons. Therefore, of them is a multiple of n, or some of the given multiples kz lies on one of
there is a hole that contains at least k them add up to a multiple of n. (From the arcs t99ll00,Ol or (0,1/100), then
+ 1 pigeons. the Third Annual Colorado Mathe- we are done-kr differs from an inte-
This simple principle works won- matical Olympiad, 1985) ger by not more than 1/100.
ders. It's amazinghow easy it is to
understdnd this idea yet how difficult
sometimes it is to discover that this
idea can be applied! AIter all, you, the
problem solver, have to createpigeon-
N holes and pigeons. The areas of appli
1, cation of the principle include num-
6
ber theory combinatorics, and geome-
try.

tigurr I tlgil't 2

0l|alllllll/tollIro
Assume now that none of the mul- we're done. integral coordinates and lies on the in-
tiples la, r = 1,2, . . . ,99, Lies on the two If the parallelogram contains three terior or perimeter of the pentagon.
arcs above. Wehave 99 pigmns (nurnben given points, then all we have left to
r, 2r, . . ., 99rlin 100 -2 = 9Spigeon- prove is a simple lemma: The maxi- 7. Forty-one rooks are placed on a
holes (the rem'aining 98 arcs). There- mum area of a triangle insuibed in a 10 x 10 chessboard. Prove that you can
fore, by the pigeonhole principle, at parallelogram of area 1 /2 isequal to choose five of them that are not at-
Ieast two of the multiples-say, 1v and 1/4. We leave the proof of this lemma tacking each other. (We say that one
tr (k > t)-lie on the same arc of length to you. rook "attacks" another if they are in
1/100. All there is left to notice is (a) We can almost hear you asking: the same row or column of the chess-
kr - tr = (k - tft is one of the given 99 "Well, is seven the smallest number board.)
multiples; (b) la- tr lies on one of the of points guaranteeing the result in Solution: Let's make a cylinder out
arcs 1991100,0) or (0, 1/100), which problems 4 and 5?" The answer is no. of the chessboard by gluing together
contradicts our assumption. The smallest number is . . . But no- two opposite sides of the board and
you try andtind it on your own. color the cylinder diagonally in 10
4. Given nine points in a triangle colors {fig. 5).
(interiorplus perimeter) of.area 1, prove Nowwehave 4l =4x 10 + l pigeons
that three of them form a triangle of (rooks) in 10 pigeonholes (one-color
area not exceeding 1/4. diagonals). Therefore, there is at least
Solution: Midlines partition the one hole containing at least 5 pigeons.
given triangle into four congruent tri- But the 5 rooks located on the same
angles of area I I a lfig.3). These con- one-color diagonal do not attack each
gruent triangles are our pigeonholes, otherl
and the given points are our pigeons. tigune 4

Now nine pigeons are sitting in four We'd like to thank Boris Dubrov, a
pigeonholes. Since 9 =2x4 + 1, there 6. Al1 vertices of a convex pentagon student at Minsk Higlr School No. 107
is at least onepigeonhole containing lie on the intersections of a grid. Prove in the USSR, for his valuable contri-
at least three pigeons. that the pentagon (interior plus pe- butions. We thank you, our readers,
If you feel that nine points are ex- rimeter) contains at least one more foryour interest andwish you happy
cessive to guarantee the result in prob- intersection of the grid. sailing through the following prob-
lem 4, you're quite right. But in order Solution: Let's introduce the coor- lems.
to prove the stronger statement in dinate system on the grid (fig. 5). To
each vertex M of the pentagon with
Addilioltal prollems
coordinates lx,ylwe assrgn the ordered
pair of the remainders upon division l. A three-dimensional space is
of coordinates (xy)by 2. There are painted in three colors. Prove that
only four possible outcomes of this there are two points exactly one mile
operation: (0,0), (0,1), (1,0), and (1,1). apart painted in the same color.
These are ourpigeonholes. Since we
have five pigeons (the vertices of the continued on page 32
pentagon), there are two vertices, M,
tigffi3 andM, that give the same pair of re-
mainders. In order to complete the
problem 5, we need to allow the pi- proof, all there is left to notice is that
geonhoies to be different in size and the midpoint of the segment MtMzhas
shape.

5. Given seven points in a triangle


of area 1, prove that three of them form \
a triangle of. areanot exceeding 1/4.
Solution: Since 7 =2x3 + 1, it \ \
would be nice to have three pigeon-
holes-then at least one of them would \ /
have at least three pigeonsl Okay,
let's draw only two midlines in the
\
I
given triangle (fig. a). We get three pi- /
geonholes. At least one of them con-
tains at least three pigeons. If one of
the triangles contains three given points, tigure 5

28 0llflrtlllll/Jarllaly 1 SS0
QaANTUM is being launched by the National Science Teachers Association (NSTA) in cooperation with the American
Association of Physics Teacher.s (AAPT) and the National Council of Teachers of Mathematia (NCTM). A board of
editors, including physicists from AAPT and mathematicians from NCTM, will provide technical review for Quantum. Jack
Wilson, Executive Officer of the American Association of Physics Teachers, and the Association's membrs and staff want
to thank everyone who has worked so hard to make this project a reality. The

American Association of Physics Teachers


organization dedicated to improving phpics education, includes student members, and invites you to
is an join! Here is some
of what AAPT has to offer to you:

o The International Physics Olympiad ;:"ffi,'ffi"


Through tests given at your school" 20 high-school students are
chosen to come to the University of Maryland at College Park
for intensive traintng and more testing. Five of those stud-ents ",.f:ft I,
are selected to attend the Olympiad, which is usually held over'
seas. In 1989, the United States had its first gold-medal winner,
Steven Gubser of Cherry Creek High School in Colorado. Steven
was one of 150 srudents ftom 30 different countries.

Soviet-United States Exchange Program


The increasing fteedom allowed Soviet citizens was reflected in
the srudent-exchange program that took place in the summer of
1989. Fifieen Soviet students came to the United States to study
and leam about our culture, while seventeen Ameican students
went to the Sovict Union. These srudents were chosenby testing
and through teacher recommendati.ons.

Contests
Several contests and awards programs for students are publi-
cized in AAPT'I news magazine, /le Announcer (which b sent
to allmembers). One award, the Metrologic High School Phys-
ics Contest, is administered by AAPT (sponsored by Metrologrc ).
This year, 30 srudent winners will receive a laser for their school.

Books, Audiovisuals, Posters, and T-shirts


AAPT has a products catalog full oJ bool<s and audiovisual aids
that make physics real and fun. Some favoites are Guilty or
Innocent? You Be a Car Crash Expert-A Physics Scenario
HypercardStack, Toys in Space,andThePuzzle of the Tacoma
Narrows Bridge Collapse For those who want to publicize their
enjoyment of physics, there are T-shins and posters. All mem-
bers receive a subsciption to The Physics Teacher, a colorful,
monthly magazine full of ideas and experiments.

For more information ahutAAFT or a free products catalog, contact:


American Association of Physics Teachers
5112 Bernyn Road
College Partq MD 20740
30u34s-4200
I

cn
q)

\
o_
e ".:fl.
LU ;,*e
o_ b
o l-
-Y E
O b
a H CU

o
o
-L
- CIT
-
s/)
Q)
IJJ L -o o
A
I- -
C
E
f,
4*
,d\,'<^,

Y -, \ (1..\
E' \\' -{
- e
q)
r(",J
*:l \
o
<./: r,?;I
E
v' 'i&

E*i eE: u
l-
i=K
xHE
g
oE,EES
* E X EH3;E!;Ei* 2 a-

F EII+:;fl ;i r-
-lEa
IEEEEgi{Eitli* E IN EE Hf E
\t
--

I E q E;E eE aui:gc€EEa*
[ct i* E; H
^-.c
-E(:
EEiEtHiE*iEgEE €s 5 EtEE D
A. O+tr ti* xr- $?Eae:;g+g.H
r ! ! O a \e 0)

x
P i€EEETF.E
(d
i\OO\\O\OcO\Od
!
C)
(n
O\OcOF-F-OoO\O
\c!f,coNNNi- E:Ee:PsE
E€€;€arE E

(!
E:EEs$eiT
f-:-- oo- un- I
oooo\ooo!
F- t/) cO oO v-
oo-.ot \d d tr
6.BKfrd
= +i+o\O\ii\O
cO\ONOLn-O\\O
(o<.cOcONcli- E 5E€ E $$Bg
:!HoiO\:l
:!oo\oGl=
:\.1 11::-
C)

x
6Ed{is}E s

5.NilXX FA #EA E rIS-H€


5 N-oi+X
coLnNoox
\d'co- eo- co- 1\ a
!
0) oh6soo r€ rEEr$fl;
nHXE{ 6 6 uuuu99

€\ I
ii x x x x x x
{s
OcO\OF-vL
X XLnLnLoLOLnto
q q -i J -.i..; -i .j
v-- rD F- Y
\OF-6lF-1
E --! !o + i + d+ 6+ ++ o+ $EEti t $s E
> oi..rt+S o 0)
AAOOOOOO E e€ H..Es'E$
o'€=8fi- ^^UUUUUU
€UUUUUUUU
gs€3d
^l\ocot=
I
3 9o
oooQoooo
6air

tiliiiii
- d o * o !EiEE c8EI
$€ E s i Es Ei
.&
6
6J
g E E c 35H
rH I I Es E3 lo
50
e
I
0)

0) I h'E J-B x.'l H.E *##Ef EE E


F H o'i= o f;'l.9.Z
H

F--dui69
H
(!


iEEiE;T#;
cO\O\fai: qEEb€
E E ;; :8 fi;Es
#gEE
\o eo co co v\ u
6iooco:l
O\Sooeo= eiE i
.= ?.:F
a C q X
rr.
>
H€€E
9 Et c, i a e :- A
Ln LO LO

o\oi.i159
Ci ''J-
.a'0 -'g q h.*t=
:IB
E€initn*E
No\F-cO\_
rn co
Lo
i^rAv
L-6
co oo
0 l'il
f.- )i
*g;o€ 3
t tr.= \u i q.J il cO O._"
tr9R-di.XFr_6=
HlelEE"oEB
qR $$i i".EH.EEEZS€E 9HLI9HAI^
e A E q EgE
\-l\OO-v- €; F E€ E€Fi-f
EE
tsg5sF
1d e.l v\ eO
<€:€EBE:tr"E !s[lcE=HH
*.a €e 3
e{dXin\
}ieoeoO\H g Fi hEbbi 3 dE
H-+e$E_ '= t+E tr !9- o5I Ep
g$FsE x
^
! c.r
\- \O eO Fr e-
:i co \o \t 6iE s;
^AyE!:>.d*Y.E b Th
E $5; EE *[E
^Y
lEoib;>J6
$5nxx
6 E H E{ E€E;*
qu
*T# H€EE:;-
il
+EEE SEEE; E
iJ]\ 'v 5
.:FOtrO.li
H O*0J
I Y.i'.
sl
d H

HAi!EiEsF
I*;
! L L ., @ u 6 A

i\,t E5g HiEE r EE E FS;E s


i,E;EEEU€:!;
iEf,P5gE.EEE E HE HE iE"HE

'gEEE* rirB
s H:g* i*- E E
ot x5g
-:.€Oe
(3;o\rtrHc\^ d;ct
- X I rr Y xY.}.,
tr\O.!?.!^^ rr Y i*
^ iE?EFE+EEiEg
s€tE EIT: T
3E!EEE$ffi eEEi+iiEEi*E, iiiEigtEii itiIiEiiig,
IN THE LAB

]luldiltu tlp under pre$$tlre


Bridges of stone, concrete-and paper
by Alexander Borovoy

IGID]TY IS ONE OF THE MOST Why are structures based on the that it doesn't unfurl. You'll see that
important properties of any engi- hollow cylinder so widespread? To the tube doesn't bend significantly
neered structure. Obviously, the find an answer to this question, we'll under the same weight. Only a sub-
structure must not change shape begin with a very simple experiment. stantially greater weight makes it de-
from the force of its own weight or flect appreciably (fig. 1b).
under the influence of the external Trils sxpeR.rvlrNr is known in the It's worth noting that we have ob-
load it must carry. literature as the "umov experiment," tained a simple method for testing
Actually, it's impossible to avoid named after the eminent Russian structures. For examplg we may evalu-
deformation altogether, but an engi- physicist N. A. Umov (184G1915). A ate the rigidity of bird feathers by
neer must choose the material and professor at Moscow University for carrying out an experiment similar to
the types of components so as to keep almost 20years, his most important that with the paper cylinder. The
deformation within a certain calcu- papers deal with the problem of en- author did it and found that a goose
lated limit. This ever-present prob- ergy transfer. In fact, he first intro- feather 10 cm long could withstand
lem for engineers is related to require- duced the concept of energy flow and the load of a 0.5-kgweight. Next, we
ments to make the structure as srmplg energy density. may apply the Umov method to so-
inexpensive, and lightweight as pos- The Umov experiment, otherwise called "sections" (fig. 2)-angle pieces,
sible, using the least amount of mate- known as "the strength of the tube, " T- and I-sectiong and comrgated sheeq
rials. In the aircraft industry, for ex- can be performed at home with the which we can make of identical sheets
ample, the balance between maximal simplest materials. Cut two rectan- of paper-and verify that they all have
rigidity and minimal mass is vitally gular sheets of thick paper, approxi- far greater rigidity than the initial
important, so engineers try to accom- mately 20 cm long and B cm wide. sheet of paper. You may carry out a
modate these mutually exclusive re- Next, using two piles of books as series of Umov experiments yoursel-f,
quirements by finding a solution that props (fig. 1), place one of the sheets on comparing the magnitude of deflec-
is in fact the optimal compromise. them and load it with small weights tion measured for different loads (or
Nature is the greatest of all inven- (if none are available/ you can use obtaining the same deflection by load-
tors. Long ago it solved a lot of prob- coins). You see that under a small ing structures with different weights).
Iems that still draw the attention of weight the sheet, which we can visu-
engineers, and rn the course of natural akze as a kind of bridge, will sag quite Now wr, NEED to understand, at
selection it created such masterpieces abit (fig. 1a). least qualitatively, what determines
as thebrd's featheg the bamboo stem, It tums out, though, that the rigid- the structure's rigidity with respect to
the hollow bones of land animals, and ity of our structure can be enhanced deflection.
so on. Notice that all these structures tenfoldby a very simple method. Let's Let's do anothff experiment. We'll
have one feature in common-they're roll the second sheet into a cylinder, or need a rectangular bar of rubber or
all tubular. tube, and wrap a thread around it so other elastic material 10 cm long with
a cross section of approximately l-2
cm2. Draw a grid of longitudinal and
transverse straight lines (fig. 3a) and
bend the bar. The grid deforms (fig. 3b)
so that the transverse lines remain
straight but are no longer parallel,
while the longitudinal lines bend. It's
easy to see that the bar's material is
subject to stress on one side and strain
Hnm I on the other. But there's a longitudi-

30 0uanlum/January I gg0
angle piece with equal sides from the neutral one. Consequently/ Another way to enhance the bridge's
to enhance the structure's rigidity, we rigidity is to use a structure called a
must place the main body of its mate- truss. You can easily make a very
rial as far as possible from the neutral simple truss on your own (fig. 4). Glue
layer. This is why the sheet of paper in a strip edgewise across the middle of a
the Umov experiment can withstand sheet of paper and connect it with taut
only a very small load while the I- threads to the ends of the sheet. (Think
section made of the same material of some way to fasten the ends of the
angle proves to be far more rigid. threads securely.) Now iI you load the
piece with But we should not be too fond of bridge, it begrns to sag and the threads
unequal sides separating the main body of material will be strained even more. Since it's
-r--t-*,.-
from the neutral layer. For example, if difficult to break the tlueads, the struc-
the I-section bar is too thin in the ture is far more rigid than the flat
middle, it becomes unstable and twists. sheet. Trusses are widely used in
On the other hand, if designed prop- building bridges (fig. 5 ) because they
erly it is four times lighter than a solid transform bending tension into stress
channel
bar with a square cross section that or strain on the bars and beams, whose
has the same rigidity. role in our experiment is played by
In many situations the hollow cyl- threads.
inder turns out to be the best type of We might also mention another
structure because of its axial symme- structure, the arched bridge lfig.6l,
try-no matter how it is loaded, it be- which has been known since ancient
haves in the same way rn every direc- times-almost as far back as the 4th
tion, and its material is far enough millennium s.c. It's based on the idea
from the neutral layer. Compared to a of transforming vertical loading into
solid cylindrical bar, the hollow one iateral compression of the arch, which
loses little in resistance to bending is transmitted downward to the bridge's
and gains a lot in material savings. For foundations.
example, if inside a solid cylinder of
diameter dwemake ahole of diame-
ter df2, the bar's rigidity will decrease
only by G7 % and approximately 25%
corrugated sheet less material will be used in construc-
tion.
tlgunc 2
Now we see why nature uses tubu-
nal line in the middle whose length lar structures so extensively. When
doesn't change. Obviously the entire life existed mainly in the oceans, there
layer of material behind this line e4pe- was no particular need for the skele- Em4
riences no deformation either. For ton to have a small mass because
this reason it's known as the "neutral buoyancy forces helped animals carry No doubt you've noticed that there
layer" (or "neutral surface"). their own weight. Some sea species- are many different kinds of bridges. In
We may infer from the experiment sharks, f or example-have inherited addition to solving engineering and
that the farther a region of the bar is massive cartilage skeletons from their economic problems when construct-
from the neutral layer, the geater the ancestors. But when animals began to ing a bridge, the architect must design
stress/ or strain, it's subject to. But, crawl on land, the skeleton's strength a beautiful shape that is in harmony
according to Hooke's law, the force of had to be combined with the lowest with its surroundings. Perhaps it's a
elastic resistance becomes greater as possible mass. Over millions of years blessing in disguise that there is no
the distance from the neutral layer cartilage tissues evolved into tubular general prescription for bridge-build-
increases, and the main contribution bones, resulting in a strong, Iight- ing. Indeed, a Russian will immedi-
to the bar's rigldir/ is from the layer far weight structure that uses material ately recail Leningrad's bridges, which
very economically.
StreSS
I
..rl-s€="+*;.
/-,*4.i++n IN oun ArrEMPrs to build apaper
\'\idffil/ bridge between two piles of books, we
leamed from experience that the bridge
must not be made flat and that shaped
straln structures as in figure 2 are more desir-
FUnI able. HUmE

0uarlrlll/lr tlls tab


present a magnificent variety of pos-
sible solutions, while an American
might thinkof the Golden Gate Bridgg
the Mackinac Bridge, or the venerable
Brooklyn Bridge.
FinaIIy, it's worth pointing out that
many famous inventors and engineers
took their first career steps by making
models of various machines and struc- .

tures. "My interest in engineering


arose during my early teens," wrote

' \E--; An arched bildge consftucted early in this century by the Swiss engineer Robert
-
Maillat, Photos andblueprin* of it canbefound in alnost any teatise on architecfixe.
The bildge overwhelms the viewu with its bold form and flawless constuction.
fumB
V. N. Obraztsov, an eminent Soviet We can see from our experiments your imagination and you make an
scientist and transportation engineer. that building a solid and beautiful interesting model, send us a photo-
"I enjoyed making models of various paper bridge, using whatever "con- graph and a description of it-perhaps
structures-not just models but scaled struction materials" come to hand, is we can share it with your fellow read-
miniature copies." not an easy task. If this activity fires ers ol Quantum. O

"Pigeonhole" from page 26 small that his footprints look like divisible by 1989 whose last four dig-
dots.) (Problem createdbyl. M. Gel- its in decimal representation arc 1990.
2. Given a square of size 1 x 1 and five fand) 12. Is there a positive integer n such
points inside it, prove that among the 7. Grandmaster Lev A1burt plays at that the last four digits of the decimal
given points there are two not more least one game of chess a day to keep representation of 3' are 0001 ?

1nu, 2ttzf2 apxt. in shape and not more than 10 games 13. Given 51 distinct two-digit num-
3. A number of people (more than one) a week to avoid tiring himself out. bers, prove that you can choose six of
came to a party. Prove that at least Prove that if he plays long enough them such that any two of the six
two of them shook equal numbers of there will be a series of consecutive numbers have distinct digits in the
hands during the party. days during which he will play ex- "ones"place and distinct digits in the
4. Prove that among any 12 distinct actLy 23 games. (From the First An- "tens" place. (Soi{er and Slobodnik)
trvodigit mrnbers there are two nrnbers nual Southampton Mathematical 14. Givenrx 10r-1+ 1 distinctk-digit
such that their difference D written in Olympiad, 1986) numbers, 0 < r < 9, prove that you can
the decimal system looks like @ 8. An organization consisting of n choose z + 1 of them such that any two
whereaisadigit. members (a, 5) has n + I three- of the r + 1 numbers in any decimal
5. Prove that among any 15 distinct member committeeq no two of which location have distinct digits. (Soifer
positive integers not exceeding 100 have identical membership. Prove andSlobodnik) O
therearefournumbers a,b, c, dsuch that there are two committees that Alexander Soifer is a pofessor of mathematics
that a + b = c + d, or there are three have exactly one member in com- at the Univercity of Colorudo qt Colorudo
numbers a, b, c formtngan arithmetic mon. (First Annual Southampton Springs. He is a fannda of the Colorudo Mathe-
progression. Mathematical Olympiad) matical Olympiad and the author of several
6. Little grooves of the same width are 9. Prove that for any 99 points located collections of math problems. His outside
interests include hiking, skiing, and at his-
dug across a long (very long!) straight in a square of area I there is a circle of tory.
road. The distance between the cen- radius L19 that contains at least *ree Edward Lozan sky is president of the Interua-
ters of any two consecutive grooves is points. tional Educational N etwork in W ashington,
2t12. Provethat no matter how narow 10. Prove that among any 5 points DC. He rcceived his master's degree fuom the
Moscow Institute of Physical Engineeilng and
the grooves aret amarlwalking along Iocated in aB x4 rectangle there are at
his Ph.D. in theorctical and mathematical
the road with a step equal to I sooner least two points not more than 5rl2 phy sics fr om the Mo scow Institute of Atomic
or later will step into a groove. (We apart. Energy. Inhis leisure time heplays piano and
assume that the man's "feet" are so 1 1. Prove that there exists an integer enioys skiing.

XZ 0urnlum/Jrnurry 1000
PROBLEM CORNER

Puzlers in lnatlt and plty$ics


Iulaflr
M1
Ilove that for every odd number a there
M5 BC,
(i1'Y"tA, f ,Ghe on the s idesAl,
D3
{trfi'en the humidity r, of airis 50%, wa-
exists a natural number b such that cA of triangle ABC and AE/EB = ter poured into a sarrcer evaporates in
2b - I is divisible by c. BF/FC : CG/GA=k, where0 <k < 1. theopenairint, =46-in. Howlong
Let K, L, M be the intersection points will it take for the water to errporat" if
M2 of the lines,4F and CE, BG and AF, CE rz= 80"/o? (A. Zilberman)
Several circles are drawn inside a unit and BG, respectively. Find the ratio of
square. Prove that if the sum of their the areas of triangles KLM and ABC. P4
circumfercnces is equal to lQ there exists (b) Use six lines to cut a triangle into thrbe uncharged capacitors of capaci-
a straight line that intersects at least four pans such that it is possible to compose tance c,, cz, c.are connected to one
circles. seven congruent triangles from them. another and to points A, B, D at poten-
(A. Soifer) tialsgo,gr, gr(fig.3). Determinethe
M3 poturtial eo atpoint O. (Ivlmcowphysics
Four ones and five zeros are written on Olympiad, 1984|
in arbitrary order. The follow-
a circle
ing operation is performed: a zero is
writtenbetween any two equal num-
bers and a one benveen anytraro distinct
p1 Physics
Figures 1 and 2 show the boundaries of
P5
Why does the presence of the ultravio-
let component inthe spectnrm decrease
numbers, then the previous numbers disturbance regions created by a ship in the sharpness of pictures obtained on
are snoved Brcve that after any number two stretches of its route. The red arrows photographic film? (L. Ashkenazy)
of such operations you will never ob- indicate the direction of the ship's travel.
tain nine zeros. There is no current in the fust stretch
(fig. 1). The direction of the current in
M4 the second stretch lfi1.2l is indicated Solutions on page 53
Every side of an equilateral triangle is by the blue arrow. Determine the
divided into r equal parts. Lines paral- cunent's velocity i{ the ship's velocity
lel to the sides of the triangie are drawn relative to the banl<s is the same inboth
through these points, thus dividing the cases-18 km/h. (V. Belonuchkin)
triangle into n2 smaller triangles. Lett
call any sequence of different triangles
achain if everytwo successive triangtes
P2
You've probably noticed that as soon

I
have a common side. \trIhat is the greatest as you set foot on wet sand, its color
possible number of triangles in a chain? becomes lighter. This is due to the fact
(M. Serov) that the sand becomes d:ryer. But as soon
as you remove your foog water imme-
diately occupies the footprint. E4ptain
this phenomenon. (L. Aslamazov)

figlm t tlgum2
A
B-D
f0uml
T"''

0mIl[il/PnotlElll Sonnen 33
TDIR.@ K Qow -recil ? tl*st . t s)
\/e E?H,?,fltia22frffiiytr',9y:,P'13, lr; 5!g:y''

f-?-T
!-r
-.,.L-s.r
- ?-
\TLL n$L LIKE
OA,G
1 Tr-tE \
crret{ ,t997L,,WPz \T ALSo A??EAQS T..t
l{\5St+SrrPt RrvER courD BE (o/rAE,
ifrN\6'tr3,fu
fo'M?P
A Hrf.t{<eEEg ARrER'(, €tNtial
25o M.pil-

K-lt[rt1r,,
E HLil

{VIVW AeZ 5u eeR co^ApuTErs, ( lE-13 lAvET.lA-r oer.lfrARcH.


AilP 5EE tF Yf cAY co*v '.

u'e wffA A SLOVI Mteta'

O,rlt\
B
\l1a M
a

Cds
34 0ualllrlll/Jarllary I 090
in which the length of the arrows rep-
resents the velocity of the liquid.) Be-
cause of intrinsic friction, the flow
velocity is different at different points
of a cross section of the capillary-it,s
greatest in the middle and decreases
toward the walls, which exert a back-

T[e $tlpsl'lluidily ot ward drag on the fluid. Measuring the


amount of liquid passing through the
capillary per unit of time, we can find
the viscosity coefficient.

helium ll The second method consists of


studying the damping of torsional
oscillations of a disk immersed in the
liquid. Here we have a physical pic-
ture very similar to the previous one.
A slippery idea The fluid is practically at rest in re-
that stuck gions far from the dis\ while the disk
exerts a drag on the layer close to it.
Different layers of fluid move at differ-
ent velocities, and internal friction
by Alexander Andreyev between them transforms the energy
of torsional vibrations into heat. We
can find the viscosity coefficient by
measuring the damping time of the
HE DrscovERyoF LreurD HruuMs lute zero). Helium exists in a solid state oscillations.
special properties was one of only at higher pressures (about 25 at- Measurements of viscosity by the
the greatest achievements of mospheres at temperatures close to ab- first method showed that the viscos-
modem physics. By now these solute zero) when the decrease in inter- ity of helium I is appreciable and
startling and paradoxical properties atomic distances enhances interaction measurable, but at transition to he-
have been unified within the con- between its atoms, resulting in solidifi- lium tr it suddenly drops to a quantity
cept of superfluidity, the phenome- cation. too small to measure, which in all
non discovered in 1938 by the Soviet At temperatures from 4.2.K to 2.2,K, Iikelihood is zero. By all appearances
physicist P. L. Kapitsa. Superiluidity liquid helium behaves in all respects as there might be a chance of consider-
doesn't reduce to a simplistic quali- an ordinary, "normal" liquid. Atz.z,It ing helium II a liquid subject to cus-
tative statement-"liquid helium helium transforms from the normal tomary laws but with very low viscos-
flows much better than any other liquid state (so-called helium I) into a ity. Measuring the viscosity of he-
liquid." As the Soviet physicist L. D. special state (helium II) possessing the lium tr with the rotating disk method,
Landau showed :u:,l94l in his theory propefty of superfluidity. In the follow- however, gives a quantity of the same
of superfluidity, the properties of su- ing pages only a few experiments with order of magnitude as that for heli-
perfluid helium are the most strik- helium tr-the most important ones- um I. Thus, in contrast to thebehav-
ing evidence of general laws that will be described. You'11see that their ior of ordinary Iiquids, helium II doesn,t
govem the behavior of any substance results utterly contradict our under- show any signs of viscosity under some
at Yery low temperatures. This is standing of the concept of ordinary liq- conditions, while under others its
why studying liquid helium has had uid and require new ideas to explain viscosity is appreciable.
such a profound in{luence on many them.
diverse fields in physics.
UiscosiU and supefl uidlU
Prorufies ol superfluid [elium In studying the viscosity, or internal
Helium is rightly said to be an friction, of helium II, we're con{ronted
inert gas. Its atoms interact very with the first puzzle of superfluidity.
weakly with other atoms/ and espe- There are two methods of determining
cially among themselves. This is the coefficient of viscosity; with ordi-
why helium turns into liquid from nary liquids they produce the same
gas at a record low temperature result.
,4.}'Kat
normal atmospheric pres- The first consists of measuring the
sure) without becoming solid at still liquid's flow through a narow capillary
lower temperatures (down to abso- under the force of gravity. (See figure 1, EUrt

0llfl[tlllll/tealllls 35
cient to produce large heat transfer. of convection in helium II is quite
Tl'anslel' ol ]leatail IlIotiolt We reach a twofold conclusion. On unusual.
In normal liquids there are two the one hand, let's assume that in
mechanisms for heat transfer: heat helium II, as in ordinary liquids, we T[e fieol'y nl sulerfluidily
flow and convection. may neglect convection when heat At room temperature certain sol-
"Heat flow" means that heat is transfer is small and assume that heat ids, liquids, and gases may exist. If we
transferred from one region to another flow plays the major role. We must increase the temperature, all liquids
exclusively because of a difference in then assign infinite thermal conduc- and solids turn into gases-that is,
temperature. To explain this more tivity to helium II. On the other hand, systems of single molecules moving
clearly, let's consider the following we may suggest that heat transfer in freely. If we increase the temperatue
experiment. A stationary heater H helium tr is always caused by convec- still further, the thermal motion of
emits energy in the direction given by the constituent atoms becomes so vio-
the arrow (fig. 21, while the liquid lent that molecules begin to decom-
remains at rest. To generate heat ]lelium Il is [olh tliscott$ pose into separate atoms. At still
transfer in this direction by heat flow, higher temperatures<f the order of a
the temperature 7, recorded on the alld 1l0lllli$c0lt$-il few tens of thousands of degrees centi-
left must behigherthanthetemperature grade-atoms decompose into elec-
depends olt hotntyotl
T, recorded on the right. Quantita- trons andnuclei. At these high tem-
tively, the rate of heat flow is deter- lll8asllr8 uistosity. peratures any substance is a gas con-
mined by thermal conductiviry which sisting of electrons and nuclei.
is the ratio of the heat to the tempera- Superfluid helium tr is a liquid that
ture difference. So for the same tem- tion and, consequently/ temperature exists only at sufficiently low tem-
perature difference there's greater heat differences are absent. peratures 12.2'K and lower). Conse-
flow from one region of the liquid to To see which of the hypotheses is quently, to explain its properties we
another when the thermal conductiv- in agreement with real life, let's put a first need to know the general laws
ity is greater. To put it the other way petal Pthat can rotate freely about a goveming the changes of thermal motion
around, a smaller temperature differ- fixed axis A rn abath of helium tr (fig. in any substance when the tempera-
ence is needed to produce the same 2). We'll find that the petal always ture is lowered.
heat flow. tums in the direction of heat transfer
"Convection" occurs when heat is whenever it occurs in helium tr. This tlemenlfl r,y Er[ilatiolt$
transferred by the actual motion of is a clear indication that motion of the Let's assume the temperature is
the fluid. Therefore, in the situation liquid accompanies heat transfer. being lowered, beginning from tens of
discussed above/ convective heat trans- Therefore, the second hypothesis is thousands of degrees centigrade. What
fer may take place at the same ther- verified. happens when electrons and a nu-
mometer readings 7, and T, iI the But the situation is far more com- cleus unite to form an atom? Before
fluid starts to move from left to right plicated than it seems at first glance. uni{ication, each of the electrons and
for some reason. Convection is gener- Let's consider the experiment Kapitsa the nucleus have three degrees of free-
ally associated with large heat trans- performed in 1941. A heater H is dom (which means they can move
fer. If heat transfer is sma1l enough, placed in a closed container partially freely in three-dimensional space), while
we can usually neglect the portion filled with helium II (fig. 3); the con- after unification only the atom as a
due to convection and determine the tainer has an outlet to a surrounding whole can move freely. Thus, the
thermal conductivity by measuring bath of helium II. The same petal P total number of degrees of freedom is
heat flow and temperature difference. with axis A is placed close to the diminished. We may saythatlower-
If we use helium II in this experi- outlet. If we turn on the heater, heat ing the temperature brings about a de-
ment, we'llfind that extremely small transfer caused by the liquid's motion crease in the possible types of thermal
differences in temperature are suffi- should flow from inside the container motion. hr fact other types of thermal
to the outside. And in fact the petal
turns to the right, indicating that he-
lium flows from the container. But
the crucial point here is that the liq-
uid's level doesn't drop during the ex-
lJ lt periment. To all appearances, the liq-
ll ll--------=-
l{ r- uid keeps flowing but the level stays
the same.
B \, This last result is convincing proof
that the motion of helium II is subject
\,
to laws different from those for ordi-
Hrut2 nary liquids. In particular, the nature Figuro 3

38 0uantum/Januany 1 SS0
motion disappear, or fteeze out, as the Iiquid in that it is accompanied by a
temperature drops still further. transition to a collective, soundlike $omepmpertissolsoundwauesinliquids
Because the motion of electrons motionof molecules as awhole. The first importantproperty of sound
relative to nuclei and of atoms relative To see this, let's recall that in solid waves in liquids is that the propaga-
to the cent'ers of mass of molecules bodies molecules can perform only tion of sound is accompanied by a
has frozen out at room temperature small oscillations at certain equilib- transfer of mass in the same direction.
(or lower temperatures), we may ig- rium sites, and the oscillations inter- In fact, if we look at the motion of a
nore the fact that molecules consist of act among themselves. In fact, oscil- particle in the liquid, we'll see that a
atoms and assume that thermal mo- lations of a molecule are immediately slow translational motion is superim-
tion is merely the motion of mole- passed on to its neighbors. As a resulg posed on its oscillations and has the
cules considered to be separate par- vibrations of the whole set of mole- same direction of propagation as the
ticles. It shouid also be noted that sound wave. But the presence of mass
when a gas or liquid becomes so1id, its transfer in a system of particles means
molecules have no means of moving Sound tltlattes are l]ts oltly that the system has a nonzero mo-
freely in space to any appreciable dis- mentum. Hence, a sound wave in a
tance and can only effect small oscil- kind ol thel'mal mnlion in
liquidhas a momentum in the direc-
lations at certain equilibrium sites.
The process of freezing out various
helium l!-all others haue tion of its propagation. This can be
verified by a number of experiments.
kinds of thermal motion also takes [een ll'ozelt oul. For example, when a sound wave hits
place, of course/ beiow room tempera- awall, momentum is an exchanged.
ture. Because all kinds of thermal The force acting on the wall is called
motion must disappear at absolute cules appear-that is, the whole solid "sound pressure."
zerot we may assume that for any body vibrates. These vibrations are Sound waves that determine ther-
substance there is a region of suffi- sound waves. mal motion usually travel in various
ciently iow temperatures in which a Of course, sound can propagate in directions and don't generate any trans-
unique kind of thermal motion-the liquids, too. h't normal liquids, though, fer of mass. In contrast, if sound
one most resistant to being frozen it fades out because its energy is trans- waves under some conditions acquire
out-persists. This kind of thermal formed into the thermal motion of a privileged direction of propagation,
motion is called "elementary excita- separate particles. Since we have a mass transfer does occur.
tion" Diffurent materia]s exhibit different liquid (helium II)in which motion of It should be noted here that in order
kinds of elementary excitation, so the single particles is frozen out, sound to emit sound, a body moving in a
main point in studying a substance at inevitably becomes the only kind of liquid at subsonic speed must vibrate.
very low temperatures is to determine thermal motion. If, however, the body is traveling at
the nature of its elementary excita- In fact, this process of freezing out supersonic speed, it can generate sound
tions. must occur at sufficiently low tem- waves (like a jet airplane breaking the
The elementary excitations of liq- peratures so that only soundlike, col- sound barrier) even though it doesn't
uid helium II are not motions of single lective motion remains. But tempera- vibrate. We can also turn the picture
helium atoms, if only because the tures that cause all other substances around: assuming the liquid morres
concept of thermal molecular motion to become solid don't lead to solidifi- and the body is at rest, we must infer
is the very basis for the theory of nor- cation of helium because there/s vir- that there is no sound emission iJ the
ma1 iiquids-which, as we have seen, tually no interaction among atoms. liquid's velocity is subsonic.
the properties of helium II obviously Consequently, the transition from in-
contradict. Therefore, the elementary dividual to coilective motion in he- An fllhltaflilr 0llelium II's Uoparlies
excitations we need are to be found lium occurs in the liquid state. Helium II's superfluidity arises di-
among other kinds of thermal mo- This properry of liquid helium singles rectly from the properties of sound
tion. Landau suggested that the ele- it out from all other fluids and as we'll waves in liquids. Infact, the friction
mentary excitations of helium II are see later, provides the basis for all the from the flow of anormal liquid througlr
collective motions of the liquid's at- r.rrusual phenomena we discussed earlier. acapillary causes the kinetic energy
oms-that is, sound vibrations; he The statement that elementary exci- of the liquid to tum into heat because
demonstrated that a consistent the- tations in helium II are sound waves of the interaction of particles of the
ory of superfluidity can be built on means the thermal motion in liquid fluid with the roughness of the walls.
this concept. helium at low temperatures is due to Since the thermal motion in helium II
There are simple arguments indi- the presence of sound waves that propa- is due to sound waves/ energy trahsfer
cating that the hlpothesis is in accord gate in ali directions. Consequently, would cause the emission of sound.
with real life. The point is that for the energy of the sound waves must But, as we have seen, this is impos-
ordinary substances the process of also increase as the intemal energy of sible at low speeds of flow.
fueeztngout the motion of single mole- the liquid increases with tempera-
cules is related to the solidificati.on o{ ture. continued on page 4A

0uolllIm/[sslure g
INNOVATORS

A.llI. l(olmouot'otl
A man of many hats
NDREY Nrrolevr,l,rcrt Kor,no- structed an integrable function with a troduced the ideas of self-similarity
gorov was one of the greatest Fourier series divergent almost every- and scaling, leading to the famous
scientists in Russian history. His where. This unexpected result cre- Kolmogorov law of2l3. Theseideas,
work in probability theory tur- ated a tremendous sensation and made and the modern developments they
bulence, and dynamic systems was Kolmogorov an intemationally recog- spawned, are now crucial elements of
fundamental and is now considered nized mathematician ovemight. statistical physics and field theory.
classic. The range of his contributions At that time, mathematics gradu- What did Kolmogorov consider his
was enormous-from poetics to stra- ate students at Moscow University most difficult achievement? His work
tigraphy, from genetics to celestial had to pass l4 examinations in vari- from 1955 to 1957 on the 13th Hilbert
mechanics, from topology to mathe- ous mathematical subjects, but it was problem, which involved the repre-
matical logic and algorithmic com- possible to substitute an original ar- sentation of continuous functions of
plexity theory. ticle on a relevant topic in piace of the many variables as the superposition of
Kolmogorov was bom on April 25, exam. Kolmogorov never took any of continuous single-variable functions
1903, in the central Russian city of the examinations, choosing instead to and on the summation operation.
Tambov. At 17 he graduated from the write the kind of papers he would Kolmogorov's last work before re-
secondary school there and entered make his life's work. Even at the tiring from active research was dedi-
the University of Moscow. Early on outset of his career, his articles con- cated to applying the ideas of informa-
he showed a keen interest in Russian tained new results in function theory, tion theory to the theory of algorithmic
history. His first work was a scientific set theory topology, mathematical logic, complexity and to the foundations of
paper on the registration of real estate probability theory, and other topics. probability theory. He proved, for
in the medieval Novgorod republic. In May 1934, a little before |ames instance, that any "computer" con-
But when he found out that history Alexander came up with the same taining N elements of fixed diameter
professors required at least five differ- idea, Kolmogorov introduced the co- related to no more than k other ele-
entproofs of every assertron, he switched homology ring, one of the most im- ments by "wires" of fixed thickness
to mathematics, where one proof suf- portant topological invariants of a space. may be packed in a cube with an edge
fices! At this time Kolmogorov found The idea came to him from physics. of approximate\y r/ N. He had guessed
himself attracted to the ancient Rus- He generalized such notions as the this result by starting from the obser-
sian arts as well, and he retained this distributions of charges and currents vation that the gray substance of the
interest for the rest of his long life. in space, on surfaces, and on lines, brain (the neurons) forms its surface,
At the age of 19 Kolmogorov con- considering the similar "functions of while the white substance (the junc-
sets" for a more abstract mathemati- tions) is inside.
cal situation. In addition to his many mathe-
Though educated in abstract, set- matical theories, Kolmogorov ex-
theoretical mathematics, Kolmogorov pounded a theory of a more human
was always interested in the natural sort: that it is impossible to do good
sciences and other applicatiors, in which mathematical research after the age
he would put aside the shackles of of 60. And so, after half a ientury of
mathematical rigor to obtain a con- original and of ten pathfinding work,
crete result. But after guessing a re- he became a high school teacher. This
sulg he invariably tried to formulate it was his main occupation for the last
rigorously as a mathematical theorem 20 years of his life. He was also ap-
or conjecture whose proof might be pointed chairman of the Commission
deduced from the fundamental postu- for Mathematical Education in the
iates of the theory. Academy of Sciences of the USSR and
Kolmogorov's work nI94I on tur- in that position instituted new pro-
bulent motions changed the face of grams to more fully develop the scien-
the theory of turbulence. Here he in- tific interests of schoolchildren.

38 0urnlum/Jrnuary I 090
In 197 0, together with I. K. Kikoyin, given off by the others. A lifetime tions. A brilliant guesser and a hard
Kolmogorov created a new magazine spent enlightening his fellow human worker, Kolmogorov was a mentor to
for Soviet youth-Kvant. He wrote beings belied this somber worldview. students and younger colleagues. And
articles for it and remained active in A. N. Kolmogorov stood out among even in his retirement Kolmogorov
managingit rightrrp until his death in the great mathematicians of the 20th nurtured yet another generation-your
t987. century in that he revolutionized both fellow readers of Quanwm in the Soviet
Kolmogorov once told a student of mathematics and physics, much as Union.
his that he thought of humanity as Newton had done two centuries ear-
individual flames wandering in a fog lier. His mind roamed freely in many L Arnold, Physics
-V. and adapted) Today
each only vaguely awareof the light fields and tirelessly sought connec- (abildged

Ride took part in a shuttle mission in


October 1984. On both flights she was
in charge of the scientific experiments

$ally Hide aboard. During her first flight, the


crew deployed and retrieved a satellite
with the shuttle's robot arm for the
Flying high on solid ground first time and conducted materials
and pharmaceutical rcsearch. Her second
flight lasted eight days, during which
HEN THE SPACE SHUTTLE the Soviet Union had been sending the crew deployed a satellite, con-
Atlantis roared into the clear women into space for years. Maybe ducted scientific observations of the
Florida sky in October, the that's why the anticipation had been earth, and demonstrated the potential
center of attention was the so great-the US space program was for satellite refuelling by astronauts.
|upiter-bound Galileo space probe. finally "catching up," as the vocabu- In |anuary 1986 Ride's training for a
Environmentalists had expressed lary of competition has it. third shuttle flight was intemrpted by
concern about the probe's nuclear Because of the fame she instantly the Challenger explosion. For the
reaetor and the risk of serious atmos- achieved on |une 18, 1983, simply by next six months, Ride served on the
pheric contamination if an accident being in orbig Sally Ride risked being Presidential Commission investigat-
occurred during takeoff or when the pigeonholed as "America's first woman ing the accident. She was then as-
probe zipped past the Earth after its in spacg" as i-f "woman in space" is a signed to NASA headquarters inWash-
boomerang trip around the sun. Many career choice. The fact is, Ride was a ington, DC, where she created the
scientists, on the other hand, were physicist before she was an astronaut Office of Exploration andproduced a
eager to see the long-delayed probe and while she was an astronaut. She is report on the future of the US space
sent on its way toward the giant planeg a physicist today. program.
about which Voyager had transmitted Sally K. Ride was bo rn onMay 26, As if taking Heisenberg's injunc-
such tantalizing information. 1951, in Encino, California. She at- tion to heart, Dr. Ride wrote a book for
Once Galileo was released, the five- tended Stanford University, eaming a children, To Space and Back, descnb-
member crew could turn to its many in Physics and a B.A. in English in
B.S. ing her experiences as a shuttle astro-
other tasks-measuring the amount 1973; anM.S.inl975; andaPh.D. in naut in simple yet evocative language.
and height distribution of ozone in the 1978. Her research in physics has In 1987 Ride left NASA to become
Earth's atmosphere, aiming a 7O-milli- focused on free-electron lasers. Seeing
meter cameta at various terrestrial that English degree tucked away in
targets, studying the causes of space that list, one is reminded of Werner
motion sickness (which afflic,ts nearly Heisenberg's remark that "even for
half of all shuttle passengers), and so the physicist the description in plain
on. language wiII be a criterion of the
Dr. Ellen Baker, a physician, con-
S. degree of understanding that has been
ducted the motion sickness tests. Her reached."
trip into space was accompanied by no Ride began astronaut training in
fanfare. She was accompanied instead 1978. At this time she was also a part-
by another woman-Dr. Shannon W. time adjunct professor of space sci-
Lucid,a biochemist. Three men and ence at Rice University in Houston.
two women in a space vehicle-no big As part of her preparation, she served
deal. as the capsule communicator at Mis-
But when Sally Ride became the sion Control for the first two shuttle
first American woman in space six missions.
years earlier, rtwas a big deal. True, In addition to her flight in 1983,

0llalllljlll/l[[oualol's 30
a Science Fellow at the Stanford Uni sity's Scripps hrstitution of Oceanog- this issue of Quantum would have
versity Center for Lrtemational Secu- raphy. ended with the suggestion that works
nty andArms Contrrl where sheworked Now that women routinely ride of art be used "when discussing the
as a physicist. In |uly 1989 she was the space shuttle to wor\ will we see laws of physics with your little
appointed director of the California more women running physics labora- brother"-period. The achievements
Space Institute and professor of phys- tories and teaching mathematics on of Sally Ride demonstrate that "little
ics at the University of Califomi4 San terua firma? That depends on those sisters," too/ are capable of reaching
Diego. As director of Cal Space, Ride who open doors-and the women who great heights in science and math.
will oversee a $3.3 million research would waik through them. Not so
institute headquartered at the univer- long ago, Albert Stasenko's article in
-TimothyWeber
" Helium ll" from page 37

Let's take a closer look at helium II motion. ,4-ll helium atoms participate components, the oscillations are damped
flowing through a capillary. At any tn both motions-th ey aren' t divided through interaction with the normal
temperature other than zero, there are into "superfluid" and "normal" at- component.
sound waves in the liquid because of orns. We may say that the superfluid
thermal motion. To study their inter- The mass of the normal compo- component shows itself in the experi-
action with the walls, let's imagine a nent increases as the energy of sound ment with the capillary flow and the
coordinate system moving with the waves increases at higher tempera- normal component in the experiment
liquid so that we can consider the tures, so that at a certain point it with the rotating disk.
liquid to be at rest while the wails equals the total mass of the liquid. Lr helium II, the process of emitting
move in the opposite direction. Su- Then superfluid motion disappears sound waves releases heat, which is
perfluidity means that the walls don't because there's no more mass to be transferred in a certain direction be-
fluid. Nonetheless,
exert a drag on the transferred, and helium II turns into cause the energy of the soundwaves in
they interact with the sound waves. helium I, whichbehaves as a normal that direction is greater than in the op-
As a result, there is an exchange of liquid capable only of normal motion. posite direction. Thereforg the direc-
momentum between them and a privi- II a body moves at subsonic speed in tion of heat transfer is at the same
leged direction for the propagation of helium II, the resulting superfluid motion time the direction of privileged propa-
the sound. Consequently, there is doesn't resist it. Inf.act, iJ the force of gation of sound waves and the related
transfer of mass in the liquid because resistance isn't equal to zerot we need mass transfer of the normal compo-
of the drag on the liquidby the walls of to use a force that performs some nent. Thus, heat transfer in helium tr
the capillary, although the mass of the work to move the body. The work can is always accompanied by a convec-
liquid involved in the motion is far only tum into heat-that is, can only tive motion of the normal compo-
less than the total mass of the liquid result in emitting sound. This, aswe nent. This is why in the experiment
because at low temperatures the en- know, is impossible. Therefore/ we shown in figure 2 the thermometer
ergy of sound waves is very small. may say there is no pressure exerted readings are always the same yet the
Wemayvisualize helium II as thougfr on the body by the superfluid compo- petal always turns in the direction of
it consists of two components that nent flowing around it. On the con- heat transfer-the normal component
can move independently of each other. trary, sound waves falling on the body's is flowing around it.
The motion of one of them isn't ac- surface exchange momentum with it [r the case of heat transfer in a con-
companied by friction; therefore, it's (as explained above in the example of tainer filled with helium II (fig. 3),
called the superfluid component. The capillary flow). So there is pressure on there is no mass transfer in one direc-
other, called the normal component/ the body from the normal component. tion because the pressure difference
exerts a drag on the walls and has Now it's not so hard to explain the due to heat transfer generates a flow of
internal friction like that of normal helium II experients described earlier. the superfluid component in the op-
liquids. The sum of the components' If we measure viscosity by the first posite direction. The velocity of su-
masses is equal to the total mass of the method (capillary flow), we don't find perfluid motion is such thaqtotal mass
Iiquid. any viscosity in helium II because the transfer is absent and the liquid's level
Of course, separating helium II superfluid component flows out very doesn't change. The deviation of the
into two components is only a man- rapidly through the leak. It doesn't petal indicates that the normal com-
ner of speaking. As we have seen, matter at all that its density is some- ponent flows out of the inner region,
there are two kinds of motion inhe- what less than the overall density but the superfluid flow remains con-
lium II, each accompaniedby its own because any normal, viscous fluid will cealed because it doesn't result in pres-
mass transfeq the sum of the masses is flow through a narrow enough leakfar sureonbodies. O
equal to the total mass of the liquid. more slowly. Measuring the viscosity Alexander Andreyev is an academician and
One motion is related to the propaga- by the rotating disk method gives a deputy diector of theVauilw Institute of Physics
Problems. He is a theoretical physicist who
tion of sound waves and is accompa- value different from zero because the works primarily in the field of solid-body and
nied by frictiory the other is superfluid disk moves in a fluid that has two low-temp er atur e phy sics.

40 [umlum/Januany l0g0
BRAI NTEASERS

Jusl lol' IhE lun ol it

BI
It's easy to show that the sum of the five acute angles of a regular star (like the These problems were proposed by AI-
ones in the American flag or the one in the Soviet flag)is 180". Prove that the
exander Korshkov (1Oth grade), A. P.
sum of the five angles of an irregular star (fig. 1 ) is also 180". Savin, A. M. Domashenko, A. A. Panov,
and B. B. Proizvolov.
p
Using each of the numbers 1,2,3, and4 twice, I succeeded in writing out an
eight-digit number inwhich there is one digit between the ones, two digits
between the twos, three digrts between the threes, and four digits between the
fours. What was the number?

Write the numbers 1 to B in the eight


circles shown in figure 2 so that any
two numbers inside circles joinedby a
Iine differbyno less than 2.
thm I

u
My grandfather's clock behaves in a strange way. During the first hal{ hour of
every hour it's 2 minutes fast, but during the second half hour it's 2 minutes
slow. How can that be explained?

85 figurs 2
An arbitrary point in-
side an equilateral tri-
angle is joined to the
three vertices and per-
pendicular lines are
dropped down to the
three sides (figure 3).
Show that the sum of
the areas of the three
red triangles equals that
of the three blue ones.

SOLUTIONS
ON PAGE 53 thune 3

0urntum/Bl,IinlBEssIs 4l
LOOKING BACK

cian and engineer Simon Stevin is dinary one," Henry the Fourth an-
working out a new system of locks swered. "Call Vidte!"
and dams, new fortresses are being That was how, on this fine autunn
constructed according to the projects day in 1594, the destinies of two very
of the mathematician Ludolph Van dissimilar men crossed.
Rouman$ Ceulen, while our mathematician
Adriaen Van Rooman has become Adriaen Van Rooman was born in
famous for his p:uzzling calculations. 1551 in the city of Leuven in the
By the wly," the envoy continued, Spanish Netherlands (now Belgium).
challBltUE "not so long ago, Van Rooman chal-
lenged all the mathematicians of the
He studied medicine and mathemat-
ics at Leuven University, where he
world. He sent a letter to many coun- obtained his doctor's degree. He was a
and Viete's triumph tries defying anyone to solve a prob- lecturer in mathematics at Leiden and
lem of his own invention. But so far no Wrirzburg universities.
one has succeeded." Van Rooman studiedgeometry and
by Yury Solovyov "The winner will certainly be a trigonometry and also dealt with prac-
Frenchman," the king laughed. tical astronomy and navigation prob-
"You Majesty," remarked the envo, lems. He worked on the problem of
"I have this letter with mg but appar- expanding the functions sin nx and
! N rHr rrnsr DAYS oF October 1594, ently France doesn't have any out- cos tlx inpowers of sin xand cos x, and
I the king of France strolled along standing mathematicians, because Van met with some success. He deter-
I the beautiful lanes of Fontaine- Rooman didn't mention a singie French- mined the numerical value of ,r to sev-
I bleau Park with the envoy from man among those to whom he ad- enteen decimal places, which was the
the Republic of the United Nether- dresses his challenge." time
greatest precision achieved at the
lands (better known as Holland-the //But nevertheless, I've got such a in Europe. During his lifetime, Van
name of its largest province). Having mathematician, and a rather extraor- Rooman was very famous in Holland
come into being as the result of a long
and persistent struggle against Span-
ish dominion, the republic was very
young-it was only in its twentieth
year. The war with Spain was still
continuing, and the Dutch govern-
ment was determined to find new
allies.
In France the flames of the long re-
ligrous civil war had recently died oug
and Henry of Navarre, overcoming
furious resistance, had just become
king of France. Henry the Fourth did
not hide his interest in Holland as an
ally in the struggle with Spain, but
above all he was interested in the rapid
growth of Dutch trade and seafaring.
For this reason/ walking in Fontaine-
bleau Park, he listened very atten-
tively to the envoy's story-about new
silk manufactures in Rotterdam, pa-
per mills in Utrecht, and shipyards in
Zandam.
"There are a lot of talented engi-
neers and scientists in HoIIand, " the
envoy recounted. "The mathemati-

42 0urntum/Janurnt 1000
and Germany, but as time went by his sible to his contemporaries, and only 11 1111
works lost their significance. Nowa- half a century after his death did they r2 34 5 6
days his name can only be found in begin to inJluence the development of 13 6 10 15 2t ...
the largest encyclopedias. algebra and geometry. t4 10 20 35 56 ...
15 15 3s 70 126 ...
Frangois Vi0te (or Franciscus Vieta Bur rrds RETURN to Fontaine- t6 2t 56 t26 252 ...
in Latin)was bom in the French town bleau. WhenVidte appeared, the en- r7 28 84 210 462 ...
of Fontaine in 1540. He began study- Yoy took out Van Rooman's letter.
ing law in 1559 but was drawn to The letter proposed solving the fol-
mathematics and astronomy. In 1571 lowing equation:
he moved to Paris, where he contin-
ued his career as a lawyer and became 45x-3795f +95634x5 Every number here is the sum of the
acquainted with Parisian mathemati- * 1138500x7 +79ll375:P number to its left and the number
cians. In 1573 ViEte startedworking - 3451207 5x11+ 1 05306075xr3 above it. It's worth mentioning that
as a counsellor of Brittany's parlia- - 28267 6280x1s + 3849 4237 5xt7 Vidte didn't express sin rx and cos nx
ment and later became Royal Privy - 48849 4l25x1e+ 48384 1 800:3t in terms of sin x and cos x/ as we do,
Counsellor to Henry III. In 1580 he - 378558800x8 + 236030652# but expressed2 sinnxand 2 cos nxin
obtained the post of Royal Reporter on - ll7 57 9 100:,,? + 469 557 00rP terms of 2 sin x and 2 cos x. If we
Requests. -14945040*1 +3754565f assurne that the values of 2 sin nx and
During the last years of Henry's -740259x35 + 111150:f7 2 cos nx, expressed in that way t are
rule, Vidte was a cipher clerk, study- -12300*e +945*r known, we get an equation of the nth
ing the correspondence between the -45#3+fs=a, degree for the unknown quantities
Spaniards and the king's enemies. He 2 sin x and 2 cos x.
discovered the key to the difficult in particular when Vidte's original aim was to find the
Spanish cipher used by Philip II, king formulas for expressing the sines of
of Spain. When Philip for.rnd out from o= l,: ls l.t f4i multiple arcs in terms of the sines of
intercepted French letters *rat his secret \'4-V G-V '8-V A the small arcs-that is, by construct-
information was being read by the To simplify the problem, Van ing tables of sines. Later these formu-
French, he complained to the pope, Rooman gave the answers for two Ias were used in algebra and geometry.
pointing out angrily that the French other values of a, expressed in a rather Lr particular, to solve the geometrical
were using sorcery and black magic cumbersomeway. problem of trisecting angle cr, Vidte
against him. After Henry III was Vidte read the letter and immedi- used the equation 3x - # = a, which is
murdered in August 1589, Vidte of- ately wrote down the answer. The satisfied by the values a = 2 srn a; x =
fered his services to Henry of Navare. envoy said there was a sealed enve- 2 sin ( cr/3). Vidte interpreted the posi-
The author of numerous papers on lope with Van Rooman's answer at tive solutions as the chords corre-
algebra, geometry trigonometry, and his residence and that he would open sponding to the arcs 2al 3 and(350" -
astronomy/ Vidte discovered the rela- it in the presence of anotary to see if 2all3. He didn't take the negative
tionship between positive roots and Vidte was right. The next day the roots into consideration at all, which
coefficients of algebraic equations, still Dutchman confirmed the correctness was accepted practice at the time.
known as the Vidte formulas. He of Vidte's answer/ andVidte, in turn, Likewise, to divide an angle into 5
found the formula for expanding the presented 22 other answers/ unknown equai parts, ViEte considered the equa-
functions sin ax and cos rix in powers to Van Rooman. In addition, ViEte tion 5x - 5:f + xs = a, which is satisfied
of sin xand cos x, and he is the author pointed out a mistake in the state- by the values a = 2 sinrx, x: 2 sin(cx/S).
of a contemporary system of algebraic ment of the problem, made by the Now it's clear howVidte succeeded
notation. His works, written in com- copyist or by Van Rooman himselJ. in solving Van Rooman's problem so
plicated language, were incomprehen- Let's try to reconstruct how Vidte quickly. He saw at once that the
found the solution for such a mon- proposed value of a is the length of the
strous (at first sight) equation. side of a regular polygoil of fifteen
To this end, we must analyze some
of his mathematical papers. His main
trigonometrical results were formu-
las for the sines and cosines of mul-
tiple arcs. Vidte obtained them in the
+t1 Fea, form of a rule for mechanically com-
puting them. It resembles the stan-
dard rule except that, instead of Pas-
cal's triangle, Vidte used the follow-
ing tahle:

0uarlrlll/Loolillg Easl 43
sides inscribedin the unit circle (check positive roots were regarded as solu- ruler, construct a circle tangent to
that!)or, which is the samg the chord tions in his time). three given circles (Apollonius's prob-
corresponding to the arc 24'. Coef.f.r- We could end here, but perhaps we lem). Vidte himself soon came up
cients for the first and subsequent should mention that the mathemati- with a beautiful geomeffic solution.
terms of the l6ft side of Van Rooman's cal competition between Vidte and It's said that, after his second de-
equation suggested that the left side is Van Rooman continued. After a whilg feat,Yan Rooman became a zealous
nothing more than the expression of Vidte proposed the following problem admirer of Vidte and even came to
2 sin 45cx,in terms of 2 sino. Since a = toVan Rooman: With compass and Paris to study under him. O
2 sin 12o, then o = 12.145 = 4.f I5,.
which means that x = 2 sin (4{15). It is
this specific solution that Vidte gave
to the Dutch envoy.
After the royal audience, Vidte
checked his supposition. Unfortu-
nately, after the necessary calcula-
tions, he discovered that the left side
0l amoeha$aillnolt
of the given equation did not coincide
with the expansion of 2 sin 45 cr in
powers of 2 sin d., corLtrary to what
A true tale of topology gone awry
he'd expected after glancing at Van
Rooman's challenge! by Alexey Sosinsky
At that moment he probably didn't
feel too good. Most likel, he felt just
awful. What had happened? Maybe Hrs rs ArRUE sroRY. It happened "With all your sophisticated alge-
there was a mistake in his exhaustive a long time ago in Moscow at the braic invariants, you're totally inca-
calculations? Apparently at that Intemational Congress of Mathe- pable of solving the simplest topologi-
moment Vidte founda totally differ- maticians. August 1966-I was cal problems," Zeeman was saying.
ent-geometric-approach to the still a post$aduate student back then. "For instance, this one."
expression of 2 sin 45 cr in terms of 2 After one of the ICM workdays, a Zeeman joined the index fingers
sin o: in order to divide the arc into 45 group of mathematicians, mostly to- and thumbs of his left and right handg
parts, it's first necessary to divide it pologists, gathered for a very inlormal forming interlocking rings (fig. 1).
into five parts, then every part into discussion. Besides the host and other "Is it possible to unlock these rings
three, and each of those into three well-known Soviet mathematicians, without moving a thumb or index
parts again. In short, the left side of the group included the outstanding finger apart?" he asked. "Can I change
Van Rooman's equation can be ob- British algebraic topologist f. F. Adams; from that position to this one"-he
tained from the system his fellow count4rynan E. C. Zeeman, unlinked his two hands and dramati-
an equally famous geometric topolo- cally moved his arms sideways and
3z-23= a, gisq and Colin Rourke, a postgraduate upwards, joining the fingers of each
3v-t'=2, student of Zeeman's. hand again but with his hands at a
5x-5# +xs=y. The conversation turned to one of distance from each other-"if my
the ageless topics in mathematical thumbs and index fingers are glued to
Only after analyzingVan Rooman's discussions-the comparative merits each other?
answers for the two other values of a of geometry and algebra. The enthusi "Of course," he continued "this is
was Vidte sure thatitwas a question of astic geom e ter Zeem anattacked the a topological problem, which means
dividing the arc into 45 parts, and he rather aloof and phlegmatic Adams, that mybody can change its shape at
corrected the mistake in the state- accusing him (and thereby all algebra- will, without any tearing, cutting, or
ment of the prrblern without any doubts. ists) of a complete lack of imagrnation gluing-like an amoeba. "
But ViBte didn't limit himself to find- and practical ineptitude. Adams didn'thurryto ariswer. The
ing one solution. The 22other solu-
tions, which he announced the next
day, took the following form:

360"k+12"
4stn 120"k+4"
= Zsrn
* 15 ,

k=1,2,...,22.
So Vidte succeeded in finding all
the positive roots (remember, only

tiguru I

4f 0urnlum/Jrnurny I 000
thut's 2

detached and somewhat bored expres- triumph to bewilderment, and he began


sion on his face hid what must have to reason out loud.
been an intense attempt to solve the "Certainly all the amoeba's trans-
problem in his mind. Quite unexpect- formations can be carried out without Nnroxnl
edly Rourke, who until then had re- taking off the jacket-all you have to Nens firr,rror E*ncH
mained modestly silent, as a graduate do is twist and stretch it in places. But
student should, joined the conversa- then after the hands are unlinked, the
tion. jacket will be wrapped around the
"Can'tbe done, sir," he said. amoeba in a very incongruous way.
This really surprised me. I knew And it won't be fair to say that I will The search ls on for tomorrow's
the correct answer to the problem- look the way I claimed I would. In problem solvers - todayl
it's fairiy well known among topolo- fact, one sleeve of the jacket is enough HIGH SCHOOL STUDENTS intgresred in
gists-and I couldn't understand how to get me all swaddled up in fabric math, science, engineering or technology
Rourke could make such an obvious can delermine their potential to succeed in
after all those transformations, so I engineering school belore applying to
blunder. Zeemarr' s surprise, however, won't be able to take the sleeve off college.
was much greater. without cuttingit. . . Colrn, I concede NEAS, A GUIDANCE ORIENTED EXAM
"You ..but Imean,"he I've lost." provides tests in Mathematical Under-
standing, Science Reading, and Problem
spluttered, "I can understand that one While a pound note was changing Solving - as well as an interest inventory,
of these algebraists . . . But youl You're hands, Adams added to Zeeman's misery out-of-class accomplishment scale and a
supposed to be a pupil of mine. This is by addressing him in biographical questionnaire.
a bored mono-
a perfectly ftivtalproblem for anyone tone: A COMPREHENSIVE SCORE REPORT
and interpretation guide to assist with high
who claims to be a geometerl// "Oh by the way, Chris, how do you school guidance etforts is sent to each stu-
But Rourke held his gound and as prove that you can't take off your dent. Transcripts are available upon
often happens when two Englishmen jacket without unlocking your fingers request.

don't agree on something/ the argu- if you don't use invariants?" REGISTER FOR THE NEAS - today's tool
for tomorrow's engineers and technicians.
ment resulted in a bet. The terms
Commended by the Engineering Deans
having been agreed upon, Zeeman As eN EXERCTsE in three-dimen- Council
immediately took a paper napkin and, sional topology, try to imagine what
S€nd for reglstratlon lnformatlon todayl
right then and there, sketched the the jacket looks like after the amoeba
required transformations of the amoeba has unlocked its fingers. Figure 3 American College Testing Program (ACT)
NEAS Registration (82)
(fig.zl. shows where the right sleeve of the P.O. Box 168
"Yes, of coursg" said Rourkg "but dinner jacket ends up. As for invari- lowa City, lowa 52243
your jacket, sir. ants-perhaps Quantum will tum to
Zeeman's expression changed from that topic in upcoming issues. CI

figunc 3

0[[[ll|lll/l.oolill0 Bacl {5
AT THE
BLACKBOARD
Don't unplug your brain just yet,
but when it comes to weeding out false assumptions . . .

Equaliolt$ Ihink lol' you


by V. Nakhshin

OLVING A COMPUTATIONAL The first two cases differ from the


problem in physics usually in- third in that they are concerned with
volves two stages: first, we kinetic (sliding) friction between the
think over the conditions of the first body and the inclined plane, whereas
problem, analyze the physical laws the third is concemed with static fric-
describing the given phenomenon, and tion. These two kinds of friction are
find the appropriate system of equa- quite different. The length of the
tions; second, we try to find an effi- vector for kinetic friction is given by
cient method for obtaining the soiu- the equationFo= FN, whereNis the
tion in general form. In a sense, we're length of the vector of normal reac-
physicists during the first part of our tion. For static friction, the only re-
work, mathematicians during the quirement is that its force be less than
second. But, having obtained the an- figurul that of kinetic friction.
swer/ we turn to physics again to see Problem l. There is a pulley at the Let's choose the first scenario. By
whether our result is reasonable-for top of a rcugh inclined plane (fig. 1). writing Newton's second law, we may
examplg whether it stands the test of Twobodiesof massesmr =3 kgand resolve the forces acting on the first
physical dimension analysis. \: 2 ks are attached to the ends of a body as those parallel to the inclined
Sometimes a review of the result rcp e slung ov er th e rim of th e pul1 ey. plane and those perpendicular to i! for
shows that it's absurd. Problems where Find the acceleration of the system the second body, we need to consider
this happens generally have a feature and the friction between the ftust body only the vertical direction. The equa-
in common: they require a carefuL and the plane if the cofficient of fric- tions read
preliminary examination of the physi tion p: 0.5 and the plane's angle of in-
cal process involved and suggest sev- clination u = 3U from thehorizontal. ?-m,g sin cl -pN = mra,
eral versions, or scenarios, that are all Three different scenarios are clearly N-mrg cos cr = 0,
reasonable enough at first sight. Ulti- possible: (1) the second body moves mz9-T:mLa.
mate success depends on a wise choice downward and the pulley rotates clock-
of the scenario, and it should be noted wise; (2) this body moves upward and It should be noted that the accelera-
that a littlebit of mathematics helps the pulley rotates counterclockwise; tion a is positive because of the choice
us make that choice. Indeed, suppose (3) the system does not move at a1l. of directions (fig. 2). But solving the
we choose a scenario, come up with a system, we get a negative a:
system of equations, solve it, and then
convince ourselves that the answer is mz-ltmlcoscr-2, sincr
a=g = -1.6 m/sec2
absurd. The conclusion is that we *r,**z
should have chosen another scenario
to analyze with oru mathematical tools. Therefore, we must reject the first
Eventually (and we can be thankful scenario.
that physical laws severely restrict Let's take a look at the second one.
the number of possible scenarios), At this point, a common mistake is to
mathematics will indicate the right set the acceleration a= 1.6 m/sec2 on
one. the grounds that we have reversed all
Let's consider a few examples. directions. But the force of friction
tigunc 2

40 0tlattltttn/Jo[[u'y 1gg0
remains negative, and the equations (I) f/r 0"C, (2) T,<O"Ci (3) Tr= 0"C. the water is cooled down to 0C, fuevr.r,,
take the form First, let's suppose 7r > 0.C-that and cools down, in the form of ice, to
is, the calorimeter will end up con- The thermal balance equation
m,gsincx-T-1tN=mra, taining only water. We can visualize
l.
then reads
N-m,g ios cr = 0, the heat transfer as follows: the mass
T-mrg=mra. m, (3 kS) of ice is heated from
I
= c
rm r(7,-T rl + c rm rl0 - T rl
-10"C to 0oC, the ice melts, the water + crmr(T,-0lr -)umr=O,
so that obtained from the ice is heated from
OoC to 7,, and the mass mr(2kgl oI which for our numerical data gives
m1't2
srno"-u,m cosct-rn
water initially contained in the calo-
a=g Tr, 0"C. Consequently, the second
=-3.6rn/secz. rimeteris
m.+m2 cooledfrom Tr:80.C to 4. hypothesis is also wrong, so we come
The thermal balance equation readd to the conclusion f = g"g.
We see that the acceleration is again At this point you may be irritated
negative, and we must reject the sec- c - T rl + ?vm r + 01 + with me and asking yourself: "Why
rm r(0 c
rm r(7,-
ond scenario. The only option left is crmr(T,-T2l=0, does he purposely select examples in
the third one-that is, a = 0. Then which a1l initial hypotheses tum out
friction is statig and the formula,Q = to be wrong and such cumbersome so-
UN doesn't hold any longer. We have lutions leadto the comparatively simple
found the solution, at least partially. results a =0, Tr= 0"C? Why doesn,t he
To find the force of friction, notice take these simple cases right off the
that the forces acting on the first body bat?"
parallel to the inclined plane are the The answer is that such an ap-
profection of the force of gravity (mrg proach is possible, but generally it
sin a, = 15 newtons) and the rope's ten- doesn't result in serious simplifica-
sion, equal to the weight of the second
t tions. For example, in the first prob-
body (T = mrg = 20 newtons). Conse- *ri lem we could assume that the system
quentl, the force of static static fric- is at rest, write out equations, solve
tion is directed parallel to the plane them, and find the force of friction.
downward: But then we have to find the force of
kinetic friction to convince our-
F, = T-mrS sin a = 5 newtons. selves that it's greater than the
static friction. I{, on the other
This is, infact,less than that of hand, for some nurnerical values
kinetic friction:
&,\ R2 R3
kinetic friction had tumed out to
be the lesser quantity, we would
PN= Pm,g cos o= 13 newtons. ,rl ,,l have to start ftom scratch.
In the second problem, if we
Ptoblem 2. Twokilograms of 62, rz had merely assumed 7,= 0"C, we
watu at temperutute +B}oC ate wouldn't have known hol,v to write
poured into a caloilmeter con- the equations of heat balance be-
taining three kilograms of ice at cause/ without making any as-
tiruru4
-10' C. What temp er ature establishes sumptions about the process taking
itself inside the calofimeter as the place in the calorimeter, we dont know
rusuh of heat ftansfer! (The heat where crl c2 represent the specific heat whether some ice melted or some
capacity of the calorimeter should of ice and of water and l, is the specific water froze. Of course, we could sug-
not be taken into account.) heat of melting. Hencg gest some way out of these difficul-
Obviously, the final temperature f, ties, but the number of possible op-
is greater than -10"C and less thari tions would be even greater than in
+80"C. The main point is that the T_ ' ,*rT ,*'r*rTr-\um, the solutions discussed above.
cr(mr+mr)
specific heat of water and that of ice
are differeng and the process of melt- However, for our numerical data T, Problem 3. Findthe currents in all
ing requires some additional heat. There- tums out to be less than OoC, so there parts of the circuit given in figure 4.
fore, the process of heat transfer essen- is a contradiction with the hypothesis HereEr= E2 = E = 1volt, 11: 12= r: 7
tially depends on whether some ice we had assumed. ohm, andR, : &= R3 : R= 70 ohms.
melts or some amount of water freezes. Suppose 4 < O'C-that is, the calo- The key to solvingthis problem is
To find out which possibility actually rimeter will contain only ice. Heat to find the right directions of the cur-
occurs/ we'l1 consider the equations of transfer should proceed as follows: rents, which determine the equations
thermal balance for three different cases: the ice is heated from -1OoC to 7.and of charge balance at the nodes, or

0uirlulll/At Tle ilacIt0art 47


branching points, of the electrical cir- where vo is the initial velocity and gis
Emcises
cuit. First, let's suppose that the cur- the free-fall acceleration. From the
rettts I 1r,1. are directed as shown in equation we inJer 1. A lever with a mass of 10 \g leans
' Electrical charges may not
figure 4. on a prop 25 cmfrom its left end. The
accumulate ai the branching points of
v*
0
vf;-zsh lever is I m long. A weight with a
I -'--- '
a circuit because incoming and outgo-
o
mass of 2 (g is suspended from the left
ing charges counterbalance each othe4 Substituting ro] zSm/sec, h = 4O m, end of the lever. What force must be
consequently, we have the equation and I = 9.81 m/secz, we obtain a nega- applied at the right end at an angle of
for the currents 1,. = I, + 1, in accor-. tive number under the square root. 30o above the horizontal so that the
dance with the hypothesis conceming Obviously, the result indicates that lever is in equilibrium?
their directions. the body will not reach this heiglrt. In 2. A 500-gram mass of steam at
Let's choose two loops inside the facg the maximum height is given by 100'C is put inside a calorimeter con-
circuit-for example, the left and the h*= vll2g=31.5 m (that is, less than tammg 1 kilogram of ice at 0'C. What
right one-and choose a certain direc- 40m). temperature results after heat transfer
tion for tracing a path-for example, So, in order to obtain reasonable has taken place? (The heat capacity of
counterclockwise. The energy con- answerg we must ask reasonable ques- the calorimeter should be ignored.)
servation principle requires the alge- tions. 3. Abody is thrown straight up at
braic sum of electromotive forces to The English naturalist T. H. Hux- an initial speed of 15 m/sec. What
be equal to the algebraic sum ofvolt- ley used to compare mathematics to height doesit attain in 2 sec? O
ages: millstones, which grind only the seeds
poured between them-nothing more.
E - E=/,R +.I,2 + Irr + IrR, The problems discussed above are ex-
-E = -IzR- lzt + IsR. amples of this general principle, but
they also teach us that, in helping us
For the numerical data of the prob- reject incorrect assumptions, mathe-
lem these equations read matics gives us a clue about the direc-
tion that will lead to a solution.
Ir= Ir+ I,
10/,+1, +Ir+l0lr=0,
l0lr+ Ir- 101. = 1,

and have the solution


Flights of Imagination
It= -t l3t A, Iz= I l3t A, Is= -2131 A. An Introduction to Aerodynamics
Note that the first and third cur- olwoys the best woy trr leorn-ond fie
rents are negative, whereas the as- most fun!
sumption we made about the direc- PB61, 56 pp., $ 7.00, middle through high
tions of the currents requires them to school. I989 Revised Edifion.
be positive. If we reverse the direc-
tions of the first and third cuffents, All orders of $25 or less
we'li obtain the right answer-three must be prepoid.
positive values of Iu Ir lr, whose abso- Orders over $25 must
Go fly o kite, ond shore with your include o purchose
lutevalues equal those obtained above.
students the excitement of seeing order. All orders must
It should be noted that the equation I8
science in oction. These revised include o postoge ond
for current conservation in this case hondling fee of $2. No
ond updoted proiects provide stu-
reads ,I, = I, + I, credits or refunds for
dents with o honds-on opprooch for
returns. Send order lo:
investigoting the lows of oerody- Publicotions Soles,
Problem 4. Abody is tfuovvn saaigfit
nomics. With trosh bogs, string, NSTA, 1742 Connecii-
up at an initial speed v o: 25 m/sec. In
dowels, ond tope, students ore en- cuiAve. NW,
what time will it reach the height h = Woshington, D.C.
couroged to try out the cleorly-
40m! 20009.
described fundomentols of flight
Let's write the equation for the
ond see how they work.
height of the body:
Whether or not oerodynomics is Learning wi,th
new to your sfudents, these proiects
The National Science
h = vot -\-"P give them the tools to onswer
questions for themselves, which is Teachers Association

48 0urnlum/JanuanI lg00
SUMMER PROGRAM
IN MATH AND SCIENCE FOR
HIGH SCHOOL STUDENTS AND TEACHERS
You are invited to participate in an exciting US - Soviet exchange program: the 1990 Science and
Mathematics Intemational Summer Institutes to be held at LaSalle Academy, Long Island; University of
Maryland, College Park; Moscow State University, USSR; and University of Tartu, Estonia.

HIGHLIGHTS OF THE PROGRAM


x Advanced mathematics, physics, computer science, and
molecular biology courses
* Russian language and literature
* Lectures by prominent scientists
* Visits to scientific laboratories
x Discussions and debates
* cultural enhancement from the international group of participants
* Excursions to New York and Washington in the US and to Leningrad and Moscow in
the USSR
* Chess, sports, sandy beaches, films, concerts, and more

The Institutes are coordinated by the National Science Teachers Association (NSTA), American Association of
Physics Teachers (AAPT), National Council of Teachers of Mathematics (NCTM), and International Educational
Network in cooperation with the Brookhaven National Laboratory and the USSR Academy of Sciences.

For more information, please fill out the coupon and mail to:
Dr. Edward D. Lozansky
NSTA
1742 Connecticut Avenue, NW
Washington, DC 20009
(202) 362-7855 or (202) 328-5800

------Please clip and mail--------

Last Name First Name


Address
City State Zip
Home Phone (

Please check if you are a high school student or teacher:

High school teacher


main subject you teach
High school student

Please send me additional brochures and application forms to circulate among teachers and students
who might be interested in panicipating in this program.

0ulnlum/JlnurnJ I 000 40
HAPPENINGS

The Tourltamenl ul Towns


A more relaxed approach to math competition
by Nikolay Konstantinov

HE TOURNAMENT OF TOWNS as well as theTournament of Towns


resembles ordinary mathemat- itself. Most of the authors are con-
ics olympiads in that high Some olile mtlllottmalnenl nected with Kvant in one way or
school students get together to another-as subscribers, readers,
solve math problems. It differs in Uollems ulill unduhlsdly authors, or editorial board members.
many ways, however, from math hecome lafl ol[te ch$sical It's traditional in the Soviet Union
olympiads as they are traditionally to send new mathematics problems
organized in the USSR. For instance, fulklnne ol mafi coln[Blition$. to Kvant, where a highly competent
olympiads are multistage competi- team of problem specialists, headed
tions at the school, city, region, re- by N.B. Vasilyev, picks out the best
public, national, and finally interna- should writeto Kvant magazine ones to use in the Tournament of
tional levels. In contrast, the Tour- (Moscow 103006, ul. Gorkogo 32/1, Towns or publish directly in Kvant's
nament of Towns is a one-day affair Kvant, Tournament of Towns) for problem section.
in which any high school student more details, competition problems, An important feature of the choice
can participate without leaving his and instructions (which need not be of problems for the tournament is
or her home town. dogmatically complied with- our that it involves two levels of compe-
The tournament takes place each traditions allow local organizers to tition-one for beginners and one
year in the spring and autumn si- modify the rules to suit local condi- for experienced problem solvers.
multaneously in many cities of the tions). Here experience is not synonymous
Soviet Union and-in the last couple I think the problems used in the with age. There are easier and harder
of years-in a few cities in Poland, competition are the most interest- problems for the two age groups
Bulgaria, and Australia as well. All ing feature of the Tournament of participating in the tournament.
these cities have their own organiz- Towns. During the 10 years the It should also be pointed out that
ing committees consisting of local tournament has been in existence, the selection of the winners is not,
educators and research mathemati- 214 problems have been proposed to by any means/ the main goal of the
cians who advertise, organize, and the competitors. These include a Tournament of Towns. Participa-
supervise the competition and cor- certain number of "practice prob- tion in the competition often helps
rect the papers. lems," usually chosen from past confirm a beginner's interest in
The central organizing commit- olympiads, but almost all the con- mathematics. In order to deempha-
tee (based in Moscow) provides the test problems are original. Some of size the purely competitive stimu-
problems and sums up the overall the new problems are real master- lus of problem solving (doing as
results. In particular, it double pieces-beautiful discoveries in much as you can within a given
checks the grades given to the best miniature that will be remembered time limit under stressful condi-
papers/ which are sent to Moscow for a long time by the contestants tionsland to avoid transforming the
by the local organizers for that pur- (whether they solved them or not) tournament into something that
pose. and will undoubtedly become part helps produce young "professional
If there is no organizing commit- of the classical folklore of math com- math problem solvers," the central
tee in your city, it's possible to have petitions. organizing committee has recently
one set up. A11 that's needed is a The authors of many of the best been sending participants "research
group of math problem enthusi- problems are past winners of vari- topics" on which they can work at
asts-teachers/ research mathema- ous contests-Moscow, Leningrad, their leisure, without the nervous
ticians, or college students-who Riga, and national math olympiads strain involved in a timed contest.

50 0uanlum/JanurnI I 890
Pl'illems lnom fie
tions hold: (1)The cardboard quadri- on each of the colored squares of Ru-
1 0ilrlburnament ol Imruns Iateral can be placed on the paper one bik's cube so as to obtain a connected
To giveyoumore specific idea of
a so that its vertices A, B, C, D lie on the line without self-intersections? (3 points)
the competition, here are the prob- "paper sidesr" one vetex on each side;
lems from the spring round, March (2) if the four visible paper triangles are
Erade$0altdl0,
t989. then folded over the cardboard quadri- A-lettel
lateral, they cover it entirely without
Enades 7 and I (age$ 13to l5), overlapprng. Prove that (a)if two quad- 1. Find two six-digit numbers whose
0.leml0egiltmr$) rilateralsfit each other, the paper one conc,atenation (the number obtained
either has two parallel sides or has per- bywriting them one after another)is
1. The positive numbers a, b, c pendicular diagonals (2 points); (b)if divisible by their product. (3 points)
satisfya >b>canda+b+ c< 1. Prove PQRS is a paper parallelogram, it's 2. The point M is chosen inside
thata2 + 3bz + ScL < l. (3 points) possible to cut out a cardboard triangle ABC so that BMC = 90" -
2. Let AMbe the median of triangle quadriJateral ABCD *rat fits it (3 points). BAC 12 and the line AM contains the
ABC. Can the radius of the incircle 4. Prove that if m is even, then the circumcenter of triangle BMC. Prove
inside triangle ABMbe exactly twice integers from 1 to m - I can be written that M is the incenter o{ triangle ABC.
thatof ACM? (3points) in sequence so that no sum of succes- (4 points)
3. What digit must be written in- sive numbers in the sequence is divis- 3. One thousand linear functions
stead of the question mark in the iblebym. (5points) are given:
number 888...88?999...99 {there are 50 5. The end points of n unit yectors f olrl = P ox * Q1,, k = l, 2, ..., l0OO.
eights and as many nines in it) in order with origin at the center of a unit Prove that to find the value of their
to make the number divisible by 7? (3 circie divide its circum{erence into n composition
points) equal parts. Some of the vectors are flxl = f ,""(f ,gsl..f ,lf ,kll- )l
4. Is it possible to draw a closed blue, the others are red. Calculate the at the point xcl no more than 30 stages
curve on the sur{ace of Rubik's cube sum of all the angles from a red vector of "parallel computations" are required.
so that it goes through each colored to a blue one (measured c,ounterclock- At each stage any arithmetical opera-
square exactly once without passing wise) and divide it by the total number tions are allowed on any pairs of numbers
through any vertex? (3 points) of all red-blue pairs of vectors. Prove obtained at the previous stages. (At
that the "mean value" of the angle the first stage the given numb ers pk,
Gnrhs 7md 8, l-lsttsl obtained in this way is always 180". (7 eo, xoare tobe used.) (5 points)
['pnolessionals') points) 4. An exclusive tennis club with 11
5. Prove that (al if Sncells of a 2n x members is ruled by a directorate
1. Astairwayhas 100steps. Nick 2n square table are marked by stars, consisting of three or more club
intends to go down the stairway in a then n rows and n columns of the table members. The club's charterforbids
special way. He starts at the top and can be crossed out so that all the stars having the same directorate twice. At
jumps down, then jumps up, then will be crossed out (4 points); (b) 3n + 1 each meeting of the club the director-
down, then up again, and so on. He stars may be placed in thetable(2nx ate must be changed-either a new
may jump 5 steps up or down (say, 2n) so that crossing out any n rows and member is added to it or an old mem-
from the ninth step to the fifteenth or any n columns leaves at least one star ber is excluded from it. Can it happen
to the third one) or 7 steps or 8 steps, not crossed out. (4 points) one day that all imaginable director-
and he may not jump onto the same ates (of three or more members) will
step twice. Will he make the descent? 0nades 0 and I 0 (l 5 and oldel'), have ruled? (6 points)
(3 points) 0-level 5. Given n lines (n > 1)ingeneralpo-
2. A pawn is placed in one of the sition (no three of them intersect at
squares o{ a chessboard (8 x 8 squares}. 1. Thepositive numbers a, b, c, d one poing no two of them are parallel)
Two players move the pawn in suc- satisfy a<b<csd and a + b + c + d>1. in a plane, prove that each of the parts
cession to any other square but each Prove that a2 + 3b2 + Scz + 7d2>l (3 into which the lines divide the plane
move (beginning with the second) must points) can be labeled by a nonzero integer
be longer than the previous one. The 2. A circle can be inscribed in whose absolute value is not greater
player who cannot make such a move ftapeziulr. ABCD. Prove that the circles than n so that the sum of the integers
loses. Who wins in an erorless game? whose diameters are the nonparallel in each of the two sides of each of the
(The pawn is always placed in the sides of the trapezium are tangent. (3 lines is equal to 0. (7 points)
exact center of a square.) (3 points) points) 6. Given 10i rectangles whose sides
3. Convex quadrilaterals ABCD 3. Find six different positive inte- are integers not greater than 100, prove
and PQRS are cut out of cardboard and gers suchthat theproduct of any two that there are three lA, B, Cl that can
paper/ respectively. Let's say that they is divisible by their sum. (3 points) be placed one inside another. (8
fit each other if the following condi- 4. Is itpossible to draw a diagonal points) o
0uanlum/fla[[enings 51
Bullelin Boal'd
We invite you to submit reports of interesting events, especially if you have taken part in them yourself and
can provide firsthand impressions. And we ask you to send us announcements of exciting things to come-math
and physics events that our readers around the country (and the world) can attend. We want to be your bulletin
board and diary-let Quantum know what's happening!

A PR0]ttlYS hom Boston Uniuensily


Ambitious high school students tation, and nuclear waste, the exhi- iletlt sounce olcomEtitiolt iilo
from all over the United States are bition wili emphasize the potential Early in 1990 the National Sci-
invited to take part in Boston Uni- for massive energy savings in the ence Teachers Association is pub-
versity's summer Program in Mathe- large-scale use of new technologies lishing a booklet on cooperating and
matics for Young Scientists (PRO- in transportation, building, lighting competing in science and math.
MYS). Students entering this six- appliances, heating, and cooling. Along with a detailed and annotated
week residential program take a Compact fluorescent lights, for ex- list of science and math competi-
challenging course in number the- ample, use only one-fourth as much tions at national and international
ory and may also study algebra. electricity as their incandescent levels, the book provides advice on
Returning students choose from cousins. organizing interest groups and start-
algebra, the theory of equations, and Further information about the fo- ing fairs. In addition, it publishes
experimental dynamical systems. rum can be obtained from the Cen- the results of surveys about contest
PROMYS emphasizes active prob- ter for Global Change, University of experiences provided by competi-
lem solving, including the formula- Maryland at College Park, Execu- tion sponsors and judges, by student
tion, criticism, and modification of tive Building, Suite 401, 7100 Balti- contestants and their teacher/men-
conjectures. Special lectures by more Avenue, CoIIege Park, MD tors, and by Nobel Prize and Medal
outside speakers heip give a broad 20740. of Science winners.
view of mathematics and its role in
the sciences.
For more information, write to It's not too early to subscribe...
PROMYS, Department of Mathe-

OUANTUM
matics, Boston University, 111
Cummington Street, Boston, MA
02215, or call 617 353-2560.

ltlloscurrr coltlergltcg to locu$


can be delivered to your door next faIl.
m e]lor'0y-efliciellcy
Scientific, political, and spiritual
Clip the couponbelow and mail to:
leaders from around the world will
gather in Moscow in |anuary for a Quantum magazine
1742 Connecticut Avenue NW
week of interdisciplinary workshops
Washington, DC 20009
and forums on critical environ-
----l
I
I

mental and development issues. The


Giobal Forum on Environment and
Development for Survival will fea- Please send one year 14 issues) of QUANTUM to: (please print)
ture such luminaries as astronomer
Carl Sagan, oceanographer )acques Name
Cousteau, and Soviet scientist
Yevgeny Velikhov. Several U.S. Address
senators are expected to attend, and
Soviet President Mikhail Gorbachev City State-Zip.
will address the 600 participants.
An exhibition of energy-efficient
technologies will serve as a center- I have enclosed a check or money order for $9.95, payable to Quantum
piece of the forum. In the context of Magazine.
global warming, acid rain, defores-

52 Illfl[lulll/Jrrllal'I I 000
SOLUT!ONS

units and zeros (and


Pl'olletn corltor both units andzeros
are actually present),
one will never obtain
M1 Nzeros. Assume the
Consider a + 1 numbers 20- l, 21- l, converse. IfNzeros
. . . ,2o - 1. Since there are only a are obtained after t
different remainders modulo a, we operations for the first
can find two of the above numbers time, then altet t- |
giving the same remainders modulo a operations you have
(the Dirichlet principle). If these num- Nequalnumbers dif-
bers are 2k - I and 2^ - I withk < m, furent from zero-that
then (2- - Il - 12x - 1) = 2r(2--r - 1) is is, Nones. It follows
divisibleby a. Since a is odd,2*- 1 is thataftert-2opera- A
divisible by a. tions everymroneiglr-
One can also prove a moregeneral boring numbers are ti[um 3
Iact: lf. a and c are coprimes, there different, which contradicts Nbeing (b) Note that rt k = 7 lz,the above ra-
exists a natural number b such that odd. tio is equal to I f 7 and each of the seg-
cb - 1 is divisible by a. ments AF, BG, CE tssplit by the two
M4 others in the ratio 3:3:1. If you draw
M2 Theansweris n2-n+ 1. Achainof this lines through points K, L, Mparallelto
Project all the circles perpendicularly Iength is shown in figure 2. To prove the segments AF, BG, CE, every side
onto some side.4B of the square (fig. 1) that chains can't be any longer, paint of triangle ABC will be split in the
and assume that every line intersects the triangles altemately in two colors ratio 2:l:l:2.
no more than three circles. Any point (as shown in the figure). It is easily Now it isn't difficult to compose 7
of the segmentAB is thus coveredby seen that there are n white triangles equal triangles from the 13 parts shown
projections of circles no more than more than colored ones. On the other (fig.3). (One of these 7 triangles is
three times. It follows that the length hand, the colors of triangles in a chain merely triangle KLMI.
of all projections is less than or equal must alternate. So there can be at
to 3. But this length is also equal to the most one white triangle more than P1
sum of the diameters of all the circles. colored ones. Thus, at least n - I white Let's combine the figures from the
Thus, the sum of the circumferences triangles do not belong to any chain. problem into a new figure (fig. a be-
of the circ.les is less than or equal to 3n. low). From an arbitrary point A on the
Since 3rc < 10, this contradicts the M5 ship's route we drop a perpendicular
original assumption that this sum is (a) Denote the area of any triangle onto the boundary of the disturbance
equal to 10. XYZby S(XYZI. Since the ratio of the region in the stretch where there is no
area of two triangles with the same curent. The length of the perpendicu-
M3 altitudes (with respect to their bases) lar (AB or AB'l determines the dis-
We'llprove an even more general state- is equal to the ratio of their bases (with tance traversedby the wave during the
ment: If there is an odd numberNof respect to their altitudes), it's easy to time the ship traveled the distance
see that AO. The distance BC (or B' C' ldeter-
S(ACK)=is(/B n=fs<een
= ,k*l slAEC)= .-l-s(A-BC).
k'+k+l k'+k+l
Similarly,

s(BLA1 = 51g 748) = "-l- S(A,BC).


k'+ k+ l
Thus,
S(KLM)
-, 3k - (t- k)2 -(t -k)3
S(ABC)' t?+tc+t t+k+t t-f
fhurs 1 figure 2

0lloIllllll/$olItio[$ 53
mines the drift of the disturbance re- packed again, the glass will not be where Ct, Cz, C, are the capacitances
gions's boundary due to the current filled to the brim. of the corresponding capacitors and
during the same interval of time. Hence, Now let's retum to the sand on the Qy, Q2, Q, ate the charges on their
theratio BC/AO lor B'C' f AO)deter- riverbank, which is also closely packed. plates. According to the law of charge
mines the ratio of the current's veloc- When pressure is exerted on the sand, conservatiorl, et+ ez+ es= 0; hence,
ity v to the ship's velocity vo. From the close packing is disturbed and the the potential of the common point O
the figure we find v/vo : I l5 :: v : 3.6 volume of the sand grows because of is
km/h. an increase in the gap between the 9o = goCr+qrCr+<PrC,
grains of sand. Waterfrom the upper C,+Cr+C.
P2 layers of sand moves deeper down,
In orderto explainwhat occurs with filling the increasing gaps. The sand P5
sand on a riverban\ we'llbegin with seems to dryout. Whenyouremove The refraction index of glass and,
some background on "close packing." your foot the close packing is restored, consequently, the focal distance of a
Identical balls can be placed on a plane and the water forced out of the re- lens depend on the wavelength of the
so that each of them touches six other duced gaps fills the footprint. radiation. By selecting the shape and
balls. We can create another layer by material of the lens and the optimum
placing balls in the spaces between P3 distance to the film plane, we practi-
balls in the first layer. Each of them Along with evaporation/ condensa- cally exclude blurring when we pho-
will touch three balls in the lower tion also occurs. The evaporation rate tograph using visible light. But when
layer and sixballs in its own layer, arrd in both cases (r, :SOY, and rr:80%)is ultraviolet rays fall on film, blurring
so on. The arrangement obtained in the same-it depends only on the occurs because the refracti.on index
this way is called close packing of the temperature of the liquid. But the for ultraviolet light differs from the
balls. If we disturb the close packing condensation rate is proportional to indexused in designing the lens. To
by takingballs in one layer out of the the concentration of vapor molecules overcome this deficiency, photogra-
in air-that is, it's proportional to the phers use filters that eliminate the ul-
relative humidity; so it's higher in the traviolet component. For the same
second case than in the first. reason some lenses have a separate
Obviously, the rate of decrease of scale marked on them for photograph-
water in the saucer is v r= v v- v clv,, v c ing in the infrared range.
are the rates of evaporation and con-
c
densation). At 100% humidity v,:v".
Taking into account all that has
been said, we may write:

V,- V | f 2 Vri
At I t = 50o/o , V rt =
at 12= B0o/o I v rr= vu-v "r= | f 5 vr.
"r=
figune 4
Since t, : tt:: v,1: v* theime trduring
spaces between balls in the lower layer, which water would evaporate at r, =
the gaps between the balls will grow. B0% is equal to
figumS
The volume of the whole system will tr=v,rtrf v,r= 5l2tr= 100 min.
also increase. This means that if a
Note: If the air convection over the surface
system of closely packed balls is acted of the liquid is insufficient, a layer oI
upon by forces leading to a distur- saturated vapor forms near the surface and Bnaintea$Er$
bance of the close packing, the vol- the rate of water decrease will be lower. Lr
ume of the systern increases because our solution we have igrrored this effect.
the gaps between the balls increase. B1
Any granular medium behaves like |oin two of the star's adjacent verti-
P4 ces-say, A and E (fig.s). Since the
this. Take, for example, millet grains Taking into account the relation be-
angles atM of triangles AME md BMD
(or coffee beans), fill a glass with them, tween the capacitance, voltage, and
are congruent/ the sum of the angles B
and shake it slightly so that the grains charge of a capacitor, we can write the
and D of triangle BMD equals the sum
form the closest possible packing. Then following equations for the three ca-
of the angles A and E of. tianf,e AME;
press the grains. The pressure will pacitors:
but then the totai sum of angles at the
cause an increase in the volume occu-
vertices of the star is equal to the sum
pied by the grains-that is, it will dis- 9a-9o =Qllc'' of the angles of triangle ACE-thatis,
turb the close packing. Some of the 9r-9o = Q2lCy 180".
grains will pour out. If we now tap the go-Qo = 4rlcy
glass so that the grains become closely

54 0urntum/Jrnuary I 000
B2 SOLUTION TO COVER PROBLEM
There are two such numbers: 41312432
and23421314. Figure 8 shows the diagonal section of the cube with inscribed spheres.
Triangles O,OrM,_and4O,D, are similar. So are triangles OrOrI\4, andAO,D,,
B3 and so on. Note also that AO1 = 31/2, OrDr = rr = 1; and O,O, = r, i r,Oror-= r,
The solution is shown in figure 6. You + r3l and so on. Therefore,
first determine that the numbers
and 8 must be put in the middle (since
1
o,o, Ao, /; oro, Ao, /-=
ol,t=qq=v r; or\= o&=v r;"'
(1)
they are each joined to 5 other circlesJ;
then the position of the numbers 2 and
7 canbe uniquely determined; then
'r*'z r:s; t3+r2 / ^J:"'
the number 3 and the others can be ,;\=v rr-rz=v 12)

fhuru 0

placed. (Obviously, the solution can


be flipped along its vertical axis, in
which case the outer pafus 3-5 and 4-6
would swap positions.)
D3 Dz Dt
B4
The clock keeps time correctl, but
the minute hand is slightly loose on FiUunc I
its spindle-it can move freely 2 min- From (2)weget
utes from its correct position. Under
the action of the force of gravity, the
r/ t-t (t/ t-t)
t/ t-t
minute hand always stays below its
t---r.l
' r/ 3+t I r --l
J l+t ' \r,/:+ t,/ -l I
----

conect position. In the left hal{ of the We can continue this process indefinitely. -llTherefore,
clock dial, this makes the clock two
minutes slow; in the right half, two
(v'T- l)'ono
minutes fast.
rqa,tJl*t)

B5 The sum of all diameters can be found by using the formula for the sum of an
If lines parallel to the sides of the inJinite geometric progression wrth the factor lql < 1; that is,
triangle are drawn through the chosen
point, pairs of congruent triangles of rl- d,
different colors are formed (figure 7). : - t-q'
where

d,= l, q
t/ l-t
t/ 3+t'
Of course, the same result can be obtained without any calculations by simply
realizing that the infinite series of diameters will converge to half of the cube,s
diagonal plus the radius of the first sphere.

0uantum/$rtutions 55
CHECKMATE!

lnuincihle tUlephi$o
A devilishly good computer chess program
byY. Gik

HE 5rH woRLD CHAMPIoNSHIP with commentary on the the more 36. Rd1-el Rb8-d8
for microcomputer chess was held interesting episodes in the games. 37. Rc6-c8 Rd8-d2
in Rome, where for the fust time 38. Bf5-h3 s7-s5
the computer crownwas decided
in tournament play. Three copies of
]lleplistu A-$p[iu I 39. c3-c4
40. c4-c5
Rd2-b2
KgS-g7
Richard Lang's renowned Mephisto, a EulM0milu 41. Rc8-c7 Nf8-h7
three-time world champion, consti- I. A-c4 e7-e5 The corect moves would have been
tuted one team; three copies of D. 2. Nbl-c3 NgS-f6 41. ... Nf8-e6l 42. Bh3xe5 Rb2xg2
Levy's less well known Sphinx made 3. Ngl-f3 Nb8-c6 stalemate.
up the other. 4. e2-e3 BfS-e7 42. f3-f4 Nh7-f6
The competition raged through six 5. d2-d4 e5xd4 43. c5-c6 Nf5-ds
rounds-the three Mephistos (A, B, 6. Nf3xd4 0-0 The activity of the black pieces
and C) played the three Sphirxes (also 7. Bfl-d3 Nc6-e5 compensates for the lack of a pawn.
A, B, and C) in two games each: white 8. e3-e4 Be7-c5 Mephisto, though, is carrying out an
and black. The results were disap- 9.8d3-e2 Bc5-b4 exchange combination, preserving its
pointing for the Sphinxes-the Me- 10. Bcl-g5 h7-h6 chances for success.
phistos won all six rounds and gar- 1 1. Bg5x{6 Bb4xc3+ 44. Rc7xf7+l Kg7xI7
nered the crown for the fourth straight l2.b2xc3 Qd8xf5 45. Bh3-e6+ KI7-f5
time. 13. Nd4-bs Qf6-g6 46. Be5xd5 RaZ-a7
Along with thebasic competition 14.0-0 Qg6xe4 47. g2-g3 Rb2-d2
in Rome another battle was taking 15. Nb5xc7 Ra8-b8 48. Bds-f3 Pta7-a2
place, a sort of "iunior world champi- 16. Nc7-b5 b7-b6 49.8f3-e4 Ra2-a4
onship." Seven programs for personal 17. Nb5-d6 Qe4-c6 50. Rel-cl Ra4xe4
computers participated in a round- 18. Qd1-d5 Ne5-g5 51. c6-c7 Re4-e8
robin toumament. The winner-Psion l9.Be2-f3 Qc6-c5 52. c7-c8Q Re8xc8
Chess; its creator-Richard Lang again. 20. Ral-dl Bc8-a6 53. Rclxc8 Rd2-d1+
It's interesting thag despite his great 21. Rfl-el Ng5-f4 54. Kg1-g2 Rdl-d2+
expertise in computer science, Lang is 22. Qd5xc5 b6xc5 55. Kg2-h3 h6-h5
anything but an "absent-minded pro- 23. Rel-e7 Rb8-b2 55. Rc8-ct Kf5-f5
fessor." Between computer competi- 24. a2-a4 Nf4-96 57. Rcl-hl Rd2-d3
tions he runs in marathons, and does 25. Re7xd7 Ng6-e5 Simplest of all would Le to force a
quite well at it. 26. Rd7xa7 Ba6xc4 draw: 57. ... g6-g5. Now white's
Here are some samples o{ com- 27. Nd5xc4 Ne5xc4 efforts are crowned with success.
puter play at the Rome tournament 28.Ra7-c7 Rb2-a2 58. Kh3-h4 Rd3-d2
29. Bf}-dS RaZxa4 s9. h2-h3 Kf5-f6

I 30. Rc7xc5
31. Bd5-b3
Nc4-b6 60. g3-g4
61. Kh4-g3 PJL-A
Rd2-f2

f,D A
Ra4-a7
32. Rc5-c6 Nb5-d7 62. Rhl-bl Rc2-c3+
33. f2-f3 Rf8-b8 53. Kg3-h4 Rc3-c5
34. Bb3-c2 Ra7-aZ 64. Rbl-b5+ Kf6-f7
E 35. Bc2-f5 Nd7-fB 65. f4-t5 g6xf5

--
50 0uanlum/Januany 1 090
66. g4-s5 Kf7-g7 Yet another blunder {24. ... Ra8-b8
67. Kh4xh5 Rc5-c7 would have been better), allowing white
68. h3-h4 Ptc7-a7 to solidly take control of the c-file.
Black resigns. White flawlessly plays through to the
end.
Plimat-Cint'us
25. Bfl-a1t. Bf7-e6
26.0-0 Rd8-d7
Sicilian0elmm
27. Rc7xd7 Be6xd7
e2-e4
1. c7-c5 28. Rfl-cl Bd7-e8
f3
2. Ng1 Nb8-c6 29.Flcl-c7 Kg8-f8
3. d2 d,l c5xd4 30. g2-g3 h7-hs
4. Nf3xdzl -\g8-i6 31.h2-h4 s7-s6
5. Nb1-c3 d7-d6 32. Kgl-f} BeB-f7
6. Bc1-g5 e7-e6 33. KI2-e3 Bf7-b3
7. Qd1-d2 a--a6 34. f3-f4 e5xf4
B. 0-0-0 Bc8-d- 35. Ke3xf4 Bb3-e5
9. f2-f4 BiE-e' 36. Kf4-g5 Be5-f7
10. Nd4-f3 b--l-i 'Ha! My mlcrocomputer could've made 37. Kg5-f5 Bf7-bB
1. e4-e5
1 br-b+ a beter move ihan thaf." 38. Kf6xg6 Ra8-e8
12. e5xf6 b-hc3 Black finally wakes up and brings
13. Qd2rc3 g-\16 the rookintoplay,but it's too late.
14. Bg;-[-t J,.-d; 39. Rc7xa7 Re8xe4
15. Kcl bl 0-0 40. Kg6xh5 Re4-a4
16. \f-l-d+ Nc6xd4 (twenty-fust game) against Korchnoi, 41. I(h5-g5 Bb3-c4
1-. Qc3rd4 Kg8-h8 Karpov chose 10. ... Rf8-e8; at Merano 42. Ra7-a8+ Kf8-e7
15. g2-g4 a6-a5 in 1981 (eleventh game), he selected 43.8a5-b7 Ra4xa8
19. g4-g5 QdB-c7 10. ... Bc5-e7. And the rook move to d8 44. Bb7xa8 Bc4-d3
20. Rd1-d3 R{8-b8 has frequently been seen in actual 45. h4-h5 Ke7-e6
21. Rd3-b3 games. The standard reaction by white 46. h5-h6 Ke6-e5
Black's position contains some dan- is 1 1. Nf3-d2, while advancement of 47.8a8-c5 Bd3-h7
gers, but it suddenly finds a way of the pawn b2-b4 has never been exam- 48. Bc6-e8 b6-b5
forcing a draw. ined in theory. Nevertheless/ the 49. Be8xb5 Ke5-e5
21. ... e6-e5l future " junior world champion" chooses 50. Bb5-e8 Ke6-e7
22. Qd4xd5 Bd7-c6 this very move, and its debut brings 51. Be8-g6 Bh7-g8
23. Qd5-c4 Qc7-dB SUCCCSS. s2.h6-h7
24. Rb3-d3 Qd8-b5 11. b2-b4l? Nc5xb4 Black resigns.
2s. Rd3-b3 Qb5-d8 12. a3xb4 Bc5xb4
26. Rb3-d3 Qd8-b6 13. Rdl-cl Nf6-e4
27. Rd3-b3 f7-f6
Chel-[emEhn
Qb5-d8 14. Bfzl-e5
Draw. 15. Be5-d4 e6-e5 lmoru{0HiU
Everything is beturg forced. Black is To close, here's an amusing ex-
PsionGhessJlimat winning back a piece and will be left ample of how a person can sti1l count
u,ith an extra pa\\,n. Yet, incredibiy, on an electronic opponent to make a
IueBtt'sGamIit
the ensr.ung endgame eventually works mistake.
t. d2-d4 Ng8-f6 out in white's favorl t. d2-d4 f7-fs
2. c2-c4 e7-e6 16.Ni3xe5 Ne4xc3 2.h2-h3 NgS-f5
3. Nbl-c3 Bf8-b4 17.Bd4xc3 f6xe5 3. g2-g4 f5xg4
4. Qdl-c2 c7-c5 18. c4xd5 Bc8-f5 4.h1xg4 Nf6xg4
5. d4xc5 0-0 19. Bc3xb4 Qa5xb4+ s. Qdl-d3 Ng4-f6
6. Ng1J3 Nb8-c5 20. Qc2-c3 Qb4xc3+ 6. Rhlxh7 Nf5xh7
7 . Bcl-f4 Bb4xc5 21. Rclxc3 Bf5-e4 7. Qd3-s5#
8. e2-e3 d7-ds 22. t2-t3 Be4xd5
9. Ral-dl QdS-a5 23. e3-e4 Bd5-f7?
t ^1, &
ST
10. a2-a3 Rf8-d8 An unforgivable error. Black had
This exchange of moves produced
a well-known position, one that oc-
curs in world championship matches.
(Bravo, computersl) At Bagio lr,1978
been confidently engaged in tactical
skirmishing ; 23. ... Bd5-c6 would have
solidi{ied its clear advantage.
24. Rc3-c7 b7-b6?
A
E E-
-!=
in Time...

rbleandamodu-
ultra-pure glass

-
.Thd,,sur{ace vessel will use

ihe museumswill',

For
a 1742 Avenue NW, W
ku;
Efi.-.S

Вам также может понравиться